Решить квадратное уравнение: Онлайн калькулятор. Решение квадратных уравнений.

Содержание

Как решать квадратные уравнения? Формулы и Примеры

Понятие квадратного уравнения

Уравнения — это математическое равенство, в котором неизвестна одна или несколько величин. Значения неизвестных нужно найти так, чтобы при их подстановке в пример получилось верное числовое равенство.

Например, возьмем выражение 3 + 4 = 7. При вычислении левой части получается верное числовое равенство, то есть 7 = 7.

Уравнением можно назвать выражение 3 + x = 7, с неизвестной переменной x, значение которой нужно найти. Результат должен быть таким, чтобы знак равенства был оправдан, и левая часть равнялась правой.

Степень уравнения можно определить по наибольшей степени, в которой стоит неизвестное. Если неизвестное стоит во второй степени — это квадратное уравнение.

Квадратное уравнение — это ax2 + bx + c = 0, где a — первый или старший коэффициент, не равный нулю, b — второй коэффициент, c — свободный член.

Чтобы запомнить месторасположение коэффициентов, давайте потренируемся определять их.

Есть три вида квадратных уравнений:

  • не имеют корней;
  • имеют один корень;
  • имеют два различных корня.

Чтобы определить, сколько корней имеет уравнение, нужно обратить внимание на дискриминант. Чтобы его найти, берем формулу: D = b2 − 4ac. А вот свойства дискриминанта:

  • если D < 0, корней нет;
  • если D = 0, есть один корень;
  • если D > 0, есть два различных корня.

С этим разобрались. А сейчас посмотрим подробнее на различные виды квадратных уравнений.

Вникать во все тонкости математической вселенной комфортнее с внимательным наставником. Наши учителя объяснят сложную тему, ответят на неловкие вопросы и вдохновят ребенка учиться. А красочная платформа с увлекательными заданиями поможет заниматься современно и в удовольствие. Запишите ребенка на бесплатный вводный урок в онлайн-школе Skysmart и попробуйте сами!


Приведенные и неприведенные квадратные уравнения

Квадратное уравнение может быть приведенным или неприведенным — все зависит от от значения первого коэффициента.

Приведенное квадратное уравнение — это уравнение, где старший коэффициент, тот который стоит при одночлене высшей степени, равен единице.

Неприведенным

называют квадратное уравнение, где старший коэффициент может быть любым.

Давайте-ка на примерах — вот у нас есть два уравнения:

  • x2 — 2x + 6 = 0
  • x2 — x — 1/4 = 0

В каждом из них старший коэффициент равен единице (которую мы мысленно представляем при x2 ), а значит уравнение называется приведенным.

  • 2x2 − 4x — 12 = 0 — первый коэффициент отличен от единицы (2), значит это неприведенное квадратное уравнение.

Каждое неприведенное квадратное уравнение можно преобразовать в приведенное, если произвести равносильное преобразование — разделить обе его части на первый коэффициент.

Запоминаем!

У преобразованного уравнения те же корни, что и у первоначального. Ну или вообще нет корней.

Пример 1. Превратим неприведенное уравнение: 8x2 + 20x — 9 = 0 — в приведенное.

Для этого разделим обе части исходного уравнения на старший коэффициент 8:

Ответ: равносильное данному приведенное уравнение x2 + 2,5x — 1,125 = 0.

Полные и неполные квадратные уравнения

В определении квадратного уравнения есть условие: a ≠ 0. Оно нужно, чтобы уравнение ax2 + bx + c = 0 было именно квадратным. Если a = 0, то уравнение обретет вид линейного: bx + c = 0.

Что касается коэффициентов b и c, то они могут быть равны нулю, как по отдельности, так и вместе. В таком случае квадратное уравнение принято назвать неполным.

Неполное квадратное уравнение —— это квадратное уравнение вида ax2 + bx + c = 0, где оба или хотя бы один из коэффициентов b и c равен нулю.

Полное квадратное уравнение — это уравнение, у которого все коэффициенты отличны от нуля.

Для самых любопытных объясняем откуда появились такие названия:
  • Если b = 0, то квадратное уравнение принимает вид ax2 + 0x+c=0 и оно равносильно ax2 + c = 0.
  • Если c = 0, то квадратное уравнение выглядит так ax2 + bx + 0 = 0, иначе его можно написать как ax2 + bx = 0.
  • Если b = 0 и c = 0, то квадратное уравнение выглядит так ax2 = 0.

Такие уравнения отличны от полного квадратного тем, что их левые части не содержат либо слагаемого с неизвестной переменной, либо свободного члена, либо и того и другого. Отсюда и их название — неполные квадратные уравнения.

Решение неполных квадратных уравнений

Как мы уже знаем, есть три вида неполных квадратных уравнений:

  • ax2 = 0, ему отвечают коэффициенты b = 0 и c = 0;
  • ax2 + c = 0, при b = 0;
  • ax2 + bx = 0, при c = 0.

Давайте рассмотрим по шагам, как решать неполные квадратные уравнения по видам.

Как решить уравнение ax

2 = 0

Начнем с решения неполных квадратных уравнений, в которых b и c равны нулю, то есть, с уравнений вида ax2 = 0.

Уравнение ax2 = 0 равносильно x2 = 0. Такое преобразование возможно, когда мы разделили обе части на некое число a, которое не равно нулю. Корнем уравнения x2 = 0 является нуль, так как 02 = 0. Других корней у этого уравнения нет, что подтверждают свойства степеней.

Таким образом, неполное квадратное уравнение ax2 = 0 имеет единственный корень x = 0.

Пример 1. Решить −6x2 = 0.

Как решаем:

  1. Замечаем, что данному уравнению равносильно x2 = 0, значит исходное уравнение имеет единственный корень — нуль.
  2. По шагам решение выглядит так:

    −6x2 = 0

    x2 = 0

    x = √0

    x = 0

Ответ: 0.

Как решить уравнение ax

2 + с = 0

Обратим внимание на неполные квадратные уравнения вида ax2 + c = 0, в которых b = 0, c ≠ 0. Мы давно знаем, что слагаемые в уравнениях носят двусторонние куртки: когда мы переносим их из одной части уравнения в другую, они надевает куртку на другую сторону — меняют знак на противоположный.

Еще мы знаем, что если обе части уравнения поделить на одно и то же число (кроме нуля) — у нас получится равносильное уравнение. Ну есть одно и то же, только с другими цифрами.

Держим все это в голове и колдуем над неполным квадратным уравнением (производим «равносильные преобразования»): ax2 + c = 0:

  • перенесем c в правую часть: ax2 = — c,
  • разделим обе части на a: x2 = — c/а.

Ну все, теперь мы готовы к выводам о корнях неполного квадратного уравнения. В зависимости от значений

a и c, выражение — c/а может быть отрицательным или положительным. Разберем конкретные случаи.

Если — c/а < 0, то уравнение x2 = — c/а не имеет корней. Все потому, что квадрат любого числа всегда равен неотрицательному числу. Из этого следует, что при — c/а < 0 ни для какого числа p равенство р2 = — c/а не является верным.

Если — c/а > 0, то корни уравнения x2 = — c/а будут другими. Например, можно использовать правило квадратного корня и тогда корень уравнения равен числу √- c/а, так как (√- c/а)2 = — c/а. Кроме того, корнем уравнения может стать -√- c/а, так как (-√- c/а)

2 = — c/а. Ура, больше у этого уравнения нет корней.

В двух словах

Неполное квадратное уравнение ax2 + c = 0 равносильно уравнению ax2 + c = 0, которое:

  • не имеет корней при — c/а < 0;
  • имеет два корня х = √- c/а и х = -√- c/а при — c/а > 0.

Пример 1. Найти решение уравнения 8x2 + 5 = 0.

Как решать:

  1. Перенесем свободный член в правую часть:

    8x2 = — 5

  2. Разделим обе части на 8:

    x2 = — 5/8

  3. В правой части осталось число со знаком минус, значит у данного уравнения нет корней.

Ответ: уравнение 8x2 + 5 = 0 не имеет корней.

Как решить уравнение ax

2 + bx = 0

Осталось разобрать третий вид неполных квадратных уравнений, когда c = 0.

Неполное квадратное уравнение ax2 + bx = 0 можно решить методом разложения на множители. Разложим на множители многочлен, который расположен в левой части уравнения — вынесем за скобки общий множитель x. Теперь можем перейти от исходного уравнения к равносильному x * (ax + b) = 0. А это уравнение равносильно совокупности двух уравнений x = 0 и ax + b = 0, последнее — линейное, его корень x = −b/a.

Таким образом, неполное квадратное уравнение ax2 + bx = 0 имеет два корня:

Пример 1. Решить уравнение 0,5x2 + 0,125x = 0

Как решать:

  1. Вынести х за скобки

    х(0,5x + 0,125) = 0

  2. Это уравнение равносильно х = 0 и 0,5x + 0,125 = 0.
  3. Решить линейное уравнение:

    0,5x = 0,125,
    х = 0,125/0,5

  4. Разделить:

    х = 0,25

  5. Значит корни исходного уравнения — 0 и 0,25.

Ответ: х = 0 и х = 0,25.

Формула Виета


Если в школьной геометрии чаще всего используется теорема Пифагора, то в школьной алгебре ведущую роль занимают формулы Виета. Теорема звучит так: 

Теорема Виета

Сумма корней x2 + bx + c = 0 равна второму коэффициенту с противоположным знаком, а произведение корней равняется свободному члену.

Если дано x2 + bx + c = 0, где x₁ и x₂ являются корнями, то справедливы два равенства:

 

Знак системы, который принято обозначать фигурной скобкой, означает, что значения x₁ и x₂ удовлетворяют обоим равенствам.

Рассмотрим теорему Виета на примере: x2 + 4x + 3 = 0.

Пока неизвестно, какие корни имеет данное уравнение. Но в соответствии с теоремой можно записать, что сумма этих корней равна второму коэффициенту с противоположным знаком. Он равен четырем, значит будем использовать минус четыре:

Произведение корней по теореме соответствует свободному члену. В данном случае свободным членом является число три. Значит:

Необходимо проверить равна ли сумма корней −4, а произведение 3. Для этого найдем корни уравнения x2 + 4x + 3 = 0. Воспользуемся формулами для чётного второго коэффициента:

Получилось, что корнями уравнения являются числа −1 и −3. Их сумма равняется второму коэффициенту с противоположным знаком, а значит решение верное.

Произведение корней −1 и −3 по теореме Виета должно равняться свободному члену, то есть числу 3. Это условие также выполняется:

Результат проделанных вычислений в том, что мы убедились в справедливости выражения:

Когда дана сумма и произведение корней квадратного уравнения, принято начинать подбор подходящих корней. Теорема, обратная теореме Виета, при таких условиях может быть главным помощником. Вот она:

Обратная теорема Виета

Если числа x1 и x2 таковы, что их сумма равна второму коэффициенту уравнения x2 + bx + c = 0, взятому с противоположным знаком, а их произведение равно свободному члену, то эти числа и есть корни x2 + bx + c = 0.

Обычно вся суть обратных теорем в том самом выводе, которое дает первая теорема. Так, при доказательстве теоремы Виета стало понятно, что сумма x1 и x2 равна −b, а их произведение равно c. В обратной теореме это и есть утверждение.

Пример 1. Решить при помощи теоремы Пифагора: x2 − 6x + 8 = 0.

Как решаем:

  1. Для начала запишем сумму и произведение корней уравнения. Сумма будет равна 6, так как второй коэффициент равен −6. А произведение корней равно 8.

  2. Когда у нас есть эти два равенства, можно подобрать подходящие корни, которые будут удовлетворять обоим равенствам системы.

    Чтобы проще подобрать корни, нужно их перемножить. Число 8 можно получить путем перемножения чисел 4 и 2 либо 1 и 8. Но значения x1 и x2 надо подбирать так, чтобы они удовлетворяли и второму равенству тоже.

    Можно сделать вывод, что значения 1 и 8 не подходят, так как они не удовлетворяют равенству x1 + x2 = 6. А значения 4 и 2 подходят обоим равенствам:

  3. Значит числа 4 и 2 — корни уравнения x2 − 6x + 8 = 0. p>

     

Дискриминант: формула корней квадратного уравнения

Чтобы найти результат квадратного уравнения, придумали формулу корней. Выглядит она так:

где D = b2 − 4ac — дискриминант квадратного уравнения.

Эта запись означает:

Чтобы легко применять эту формулу, нужно понять, как она получилась. Давайте разбираться.

Выводим формулу корней квадратного уравнения

Продолжим изучать формулу корней квадратного уравнения.

Пусть перед нами есть задача решить квадратное уравнение ax2 + bx + c = 0. Выполним ряд равносильных преобразований:

Так, мы пришли к уравнению , которое полностью равносильно исходному ax2 + bx + c = 0.

Отсюда выводы про корни уравнения :

И еще один вывод: есть у уравнения корень или нет, зависит от знака выражения в правой части. При этом важно помнить, что знак этого выражения задается знаком числителя. Потому выражение принято называть дискриминантом квадратного уравнения и обозначается буквой D.

По значению и знаку дискриминанта можно сделать вывод, есть ли действительные корни у квадратного уравнения, и сколько.

Повторим:

Алгоритм решения квадратных уравнений по формулам корней

Теперь мы знаем, что при решении квадратных уравнения можно использовать универсальную формулу корней — это помогает находить комплексные корни.

В 8 классе на алгебре можно встретить задачу по поиску действительных корней квадратного уравнения. Для этого важно перед использованием формул найти дискриминант и убедиться, что он неотрицательный, и только после этого вычислять значения корней. Если дискриминант отрицательный, значит уравнение не имеет действительных корней.

Алгоритм решения квадратного уравнения ax2 + bx + c = 0:

  • вычислить его значение дискриминанта по формуле D = b2−4ac;
  • если дискриминант отрицательный, зафиксировать, что действительных корней нет;
  • если дискриминант равен нулю, вычислить единственный корень уравнения по формуле х = — b2/2a;
  • если дискриминант положительный, найти два действительных корня квадратного уравнения по формуле корней

Чтобы запомнить алгоритм решения квадратных уравнений и с легкостью его использовать, давайте тренироваться!

Примеры решения квадратных уравнений

Как решать квадратные уравнения мы уже знаем, осталось закрепить знания на практике.

Пример 1. Решить уравнение −4x2 + 28x — 49 = 0.

Как решаем:

  1. Найдем дискриминант: D = 282 — 4(-4)(-49) = 784 — 784 = 0
  2. Так как дискриминант равен нулю, значит это квадратное уравнение имеет единственный корень
  3. Найдем корень

    х = — 28/2(-4)

    х = 3,5

Ответ: единственный корень 3,5.

Пример 2. Решить уравнение 54 — 6x2 = 0.

Как решаем:

  1. Произведем равносильные преобразования. Умножим обе части на −1

    54 — 6x2 = 0 | *(-1)

    6x2 — 54 = 0

  2. Оставим неизвестное в одной части, остальное перенесем с противоположным знаком в другую

    6x2 = 54

    х2 = 9

    х = ±√9

    х1 = 3, х2 = — 3

Ответ: два корня 3 и — 3.

Пример 3. Решить уравнение x2— х = 0.

Как решаем:

  1. Преобразуем уравнение так, чтобы появились множители

    х(х — 1) = 0

    х₁ = 0, х₂ = 1

Ответ: два корня 0 и 1.

Пример 4. Решить уравнение x2— 10 = 39.

Как решаем:

  1. Оставим неизвестное в одной части, остальное перенесем с противоположным знаком в другую

    x2— 10 = 39

    x2= 39 + 10

    x2= 49

    х = ±√49

    х₁ = 7, х₂ = −7

Ответ: два корня 7 и −7.

Пример 5. Решить уравнение 3x2— 4x+94 = 0.

Как решаем:

  1. Найдем дискриминант по формуле

    D = (-4)2 — 4 * 3 * 94 = 16 — 1128 = −1112

  2. Дискриминант отрицательный, поэтому корней нет.

Ответ: корней нет.

В школьной программе за 8 класс нет обязательного требования искать комплексные корни, но такой подход может ускорить ход решения. Если дискриминант отрицательный — сразу пишем ответ, что действительных корней нет и не мучаемся.

Приходите решать примеры на бытовых ситуациях, с красочными героями и в интерактивном формате.

Запишите вашего ребенка на бесплатный пробный урок в онлайн-школу Skysmart: познакомимся, покажем, как все устроено на платформе и наметим вдохновляющую программу обучения.

Формула корней для четных вторых коэффициентов

Рассмотрим частный случай. Формула решения корней квадратного уравнения , где D = b2 — 4ac, помогает получить еще одну формулу, более компактную, при помощи которой можно решать квадратные уравнения с четным коэффициентом при x. Рассмотрим, как появилась эта формула.

Например, нам нужно решить квадратное уравнение ax2 + 2nx + c = 0. Сначала найдем его корни по известной нам формуле. Вычислим дискриминант D = (2n)2— 4ac = 4n2 — 4ac = 4(n2— ac) и подставим в формулу корней:

Для удобства вычислений обозначим выражение n2 -ac как D1. Тогда формула корней квадратного уравнения со вторым коэффициентом 2·n примет вид:

где D1 = n2— ac.

Самые внимательные уже заметили, что D = 4D1, или D1= D/4. Проще говоря, D1 — это четверть дискриминанта. И получается, что знак D1 является индикатором наличия или отсутствия корней квадратного уравнения.

Сформулируем правило. Чтобы найти решение квадратного уравнения со вторым коэффициентом 2n, нужно:

  • вычислить D1= n2— ac;
  • если D1< 0, значит действительных корней нет;
  • если D1= 0, значит можно вычислить единственный корень уравнения по формуле;
  • если же D1> 0, значит можно найти два действительных корня по формуле

Упрощаем вид квадратных уравнений

Если мы ходили в школу всегда одной тропинкой, а потом вдруг обнаружили путь короче — это значит теперь у нас есть выбор: упростить себе задачу и сократить время на дорогу или прогуляться по привычному маршруту.

Так же и при вычислении корней квадратного уравнения. Ведь проще посчитать уравнение 11x2 — 4 x — 6 = 0, чем 1100x2 — 400x — 600 = 0.

Часто упрощение вида квадратного уравнения можно получить через умножение или деление обеих частей на некоторое число. Например, в предыдущем абзаце мы упростили уравнение 1100x2 — 400x — 600 = 0, просто разделив обе части на 100.

Такое преобразование возможно, когда коэффициенты не являются взаимно простыми числами. Тогда принято делить обе части уравнения на наибольший общий делитель абсолютных величин его коэффициентов.

Покажем, как это работает на примере 12x2— 42x + 48 = 0. Найдем наибольший общий делитель абсолютных величин его коэффициентов: НОД (12, 42, 48) = 6. Разделим обе части исходного квадратного уравнения на 6, и придем к равносильному уравнению 2x2 — 7x + 8 = 0. Вот так просто.

А умножение обеих частей квадратного уравнения отлично помогает избавиться от дробных коэффициентов. Умножать в данном случае лучше на наименьшее общее кратное знаменателей его коэффициентов. Например, если обе части квадратного уравнения

умножить на НОК (6, 3, 1) = 6, то оно примет более простой вид x2 + 4x — 18 = 0.

Также для удобства вычислений можно избавиться от минуса при старшем коэффициенте квадратного уравнения — для этого умножим или разделим обе части на −1. Например, удобно от квадратного уравнения −2x2— 3x + 7 = 0 перейти к решению 2x2 + 3x — 7 = 0.

Связь между корнями и коэффициентами

Мы уже запомнили, что формула корней квадратного уравнения выражает корни уравнения через его коэффициенты:

Из этой формулы, можно получить другие зависимости между корнями и коэффициентами.

Например, можно применить формулы из теоремы Виета:

  • x₁ + x₂ = — b/a,
  • x₁* x₂ = c/a.

Для приведенного квадратного уравнения сумма корней равна второму коэффициенту с противоположным знаком, а произведение корней — свободному члену. Например, по виду уравнения 3x2— 7x + 22 = 0 можно сразу сказать, что сумма его корней равна 7/3, а произведение корней равно 22/3.

Можно активно использовать уже записанные формулы и с их помощью получить ряд других связей между корнями и коэффициентами квадратного уравнения. Таким образом можно выразить сумму квадратов корней квадратного уравнения через его коэффициенты:

А еще найти корни квадратного уравнения можно с помощью онлайн-калькулятора. Пользуйтесь им, если уже разобрались с темой и щелкаете задачки легко и без помощников:

Квадратное уравнение

Предварительные навыки

Что такое квадратное уравнение и как его решать?

Мы помним, что уравнение это равенство, содержащее в себе переменную, значение которой нужно найти.

Если переменная, входящая в уравнение, возведенá во вторую степень (в квадрат), то такое уравнение называют уравнением второй степени или квадратным уравнением.

Например, следующие уравнения являются квадратными:

Решим первое из этих уравнений, а именно x− 4 = 0.

Все тождественные преобразования, которые мы применяли при решении обычных линейных уравнений, можно применять и при решении квадратных.

Итак,  в уравнении x− 4 = 0 перенесем член −4 из левой части в правую часть, изменив знак:

Получили уравнение x= 4. Ранее мы говорили, что уравнение считается решённым, если в одной части переменная записана в первой степени и её коэффициент равен единице, а другая часть равна какому-нибудь числу. То есть чтобы решить уравнение, его следует привести к виду x = a, где a — корень уравнения.

У нас переменная x всё ещё во второй степени, поэтому решение необходимо продолжить.

Чтобы решить уравнение x= 4, нужно ответить на вопрос при каком значении x левая часть станет равна 4. Очевидно, что при значениях 2 и −2. Чтобы вывести эти значения воспользуемся определением квадратного корня.

Число b называется квадратным корнем из числа a, если b= a и обозначается как

У нас сейчас похожая ситуация. Ведь, что такое x= 4? Переменная x в данном случае это квадратный корень из числа 4, поскольку вторая степень x прирáвнена к 4.

Тогда можно записать, что . Вычисление правой части позвóлит узнать чему равно x. Квадратный корень имеет два значения: положительное и отрицательное. Тогда получаем = 2 и = −2.

Обычно записывают так: перед квадратным корнем ставят знак «плюс-минус», затем находят арифметическое значение квадратного корня. В нашем случае на этапе когда записано выражение , перед следует поставить знак ±

Затем найти арифметическое значение квадратного корня

Выражение = ± 2 означает, что = 2 и = −2. То есть корнями уравнения x− 4 = 0 являются числа 2 и −2. Запишем полностью решение данного уравнения:

Выполним проверку. Подставим корни 2 и −2 в исходное уравнение и выполним соответствующие вычисления. Если при значениях 2 и −2 левая часть равна нулю, то это будет означать, что уравнение решено верно:

В обоих случаях левая часть равна нулю. Значит уравнение решено верно.

Решим ещё одно уравнение. Пусть требуется решить квадратное уравнение (+ 2)= 25

Для начала проанализируем данное уравнение. Левая часть возведенá в квадрат и она равна 25. Какое число в квадрате равно 25? Очевидно, что числа 5 и −5

То есть наша задача найти x, при которых выражение + 2 будет равно числам 5 и −5. Запишем эти два уравнения:

Решим оба уравнения. Это обычные линейные уравнения, которые решаются легко:

Значит корнями уравнения (+ 2)= 25 являются числа 3 и −7.

В данном примере как и в прошлом можно использовать определение квадратного корня. Так, в уравнения (+ 2)= 25 выражение (+ 2) представляет собой квадратный корень из числа 25. Поэтому можно cначала записать, что .

Тогда правая часть станет равна ±5. Полýчится два уравнения: + 2 = 5 и + 2 = −5. Решив по отдельности каждое из этих уравнений мы придём к корням 3 и −7.

Запишем полностью решение уравнения (+ 2)= 25

Из рассмотренных примеров видно, что квадратное уравнение имеет два корня. Чтобы не забыть о найденных корнях, переменную x можно подписывать нижними индексами. Так, корень 3 можно обозначить через x1, а корень −7 через x2

В предыдущем примере тоже можно было сделать так. Уравнение x− 4 = 0 имело корни 2 и −2. Эти корни можно было обозначить как x= 2 и x= −2. 

Бывает и так, что квадратное уравнение имеет только один корень или вовсе не имеет корней. Такие уравнения мы рассмотрим позже.

Сделаем проверку для уравнения (+ 2)= 25. Подставим в него корни 3 и −7. Если при значениях 3 и −7 левая часть равна 25, то это будет означать, что уравнение решено верно:

В обоих случаях левая часть равна 25. Значит уравнение решено верно.

Квадратное уравнение бывает дано в разном виде. Наиболее его распространенная форма выглядит так:

ax2 + bx + c = 0,
где a, b, c — некоторые числа, x — неизвестное.

Это так называемый общий вид квадратного уравнения. В таком уравнении все члены собраны в общем месте (в одной части), а другая часть равна нулю. По другому такой вид уравнения называют нормальным видом квадратного уравнения.

Пусть дано уравнение 3x+ 2= 16. В нём переменная x возведенá во вторую степень, значит уравнение является квадратным. Приведём данное уравнение к общему виду.

Итак, нам нужно получить уравнение, которое будет похоже на уравнение axbx = 0. Для этого в уравнении 3x+ 2= 16 перенесем 16 из правой части в левую часть, изменив знак:

3x2 + 2x − 16 = 0

Получили уравнение 3x+ 2− 16 = 0. В этом уравнении = 3, = 2, = −16.

В квадратном уравнении вида axbx = 0 числа a, b и c имеют собственные названия. Так, число a называют первым или старшим коэффициентом; число b называют вторым коэффициентом; число c называют свободным членом.

В нашем случае для уравнения 3x+ 2− 16 = 0 первым или старшим коэффициентом является 3; вторым коэффициентом является число 2;  свободным членом является число −16. Есть ещё другое общее название для чисел a, b и c — параметры.

Так, в уравнении 3x+ 2− 16 = 0 параметрами являются числа 3, 2 и −16.

В квадратном уравнении желательно упорядочивать члены так, чтобы они располагались в таком же порядке как у нормального вида квадратного уравнения.

Например, если дано уравнение −5 + 4x= 0, то его желательно записать в нормальном виде, то есть в виде ax2+ bx + c = 0.

В уравнении −5 + 4xx = 0 видно, что свободным членом является −5, он должен располагаться в конце левой части. Член 4x2 содержит старший коэффициент, он должен располагаться первым. Член x соответственно будет располагаться вторым:

Квадратное уравнение в зависимости от случая может принимать различный вид. Всё зависит от того, чему равны значения a, b и с.

Если коэффициенты a, b и c не равны нулю, то квадратное уравнение называют полным. Например, полным является квадратное уравнение 2x+ 6x − 8 = 0.

Если какой-то из коэффициентов равен нулю (то есть отсутствует), то уравнение значительно уменьшается и принимает более простой вид. Такое квадратное уравнение называют неполным. Например, неполным является квадратное уравнение 2x+ 6= 0, в нём имеются коэффициенты a и b (числа 2 и 6), но отсутствует свободный член c.

Рассмотрим каждый из этих видов уравнений, и для каждого из этих видов определим свой способ решения.

Пусть дано квадратное уравнение 2x+ 6x − 8 = 0. В этом уравнении = 2, = 6, = −8. Если b сделать равным нулю, то уравнение примет вид:

Получилось уравнение 2x− 8 = 0. Чтобы его решить перенесем −8 в правую часть, изменив знак:

2x= 8

Для дальнейшего упрощения уравнения воспользуемся ранее изученными тождественными преобразованиями. В данном случае можно разделить обе части на 2

У нас получилось уравнение, которое мы решали в начале данного урока. Чтобы решить уравнение x= 4, следует воспользоваться определением квадратного корня. Если x= 4, то . Отсюда = 2 и = −2.

Значит корнями уравнения 2x− 8 = 0 являются числа 2 и −2. Запишем полностью решение данного уравнения:

Выполним проверку. Подставим корни 2 и −2 в исходное уравнение и выполним соответствующие вычисления. Если при значениях 2 и −2 левая часть равна нулю, то это будет означать, что уравнение решено верно:

В обоих случаях левая часть равна нулю, значит уравнение решено верно.

Уравнение, которое мы сейчас решили, является неполным квадратным уравнением. Название говорит само за себя. Если полное квадратное уравнение выглядит как axbx = 0, то сделав коэффициент b нулём получится неполное квадратное уравнение ax= 0.

У нас тоже сначала было полное квадратное уравнение 2x+ 6− 4 = 0. Но мы сделали коэффициент b нулем, то есть вместо числа 6 поставили 0. В результате уравнение обратилось в неполное квадратное уравнение 2x− 4 = 0.

В начале данного урока мы решили квадратное уравнение x− 4 = 0. Оно тоже является уравнением вида ax= 0, то есть неполным. В нем = 1, = 0, с = −4.

Также, неполным будет квадратное уравнение, если коэффициент c равен нулю.

Рассмотрим полное квадратное уравнение 2x+ 6x − 4 = 0. Сделаем коэффициент c нулём. То есть вместо числа 4 поставим 0

Получили квадратное уравнение 2x+ 6x=0, которое является неполным. Чтобы решить такое уравнение, переменную x выносят за скобки:

Получилось уравнение x(2+ 6) = 0 в котором нужно найти x, при котором левая часть станет равна нулю. Заметим, что в этом уравнении выражения x и (2+ 6) являются сомножителями. Одно из свойств умножения говорит, что произведение равно нулю, если хотя бы один из сомножителей равен нулю (или первый сомножитель или второй).

В нашем случае равенство будет достигаться, если x будет равно нулю или (2+ 6) будет равно нулю. Так и запишем для начала:

Получилось два уравнения: = 0 и 2+ 6 = 0. Первое уравнение решать не нужно — оно уже решено. То есть первый корень равен нулю.

Чтобы найти второй корень, решим уравнение 2+ 6 = 0. Это обычное линейное уравнение, которое решается легко:

Видим, что второй корень равен −3.

Значит корнями уравнения 2x+ 6= 0 являются числа 0 и −3. Запишем полностью решение данного уравнения:

Выполним проверку. Подставим корни 0 и −3 в исходное уравнение и выполним соответствующие вычисления. Если при значениях 0 и −3 левая часть равна нулю, то это будет означать, что уравнение решено верно:

Следующий случай это когда числа b и с равны нулю. Рассмотрим полное квадратное уравнение 2x+ 6− 4 = 0. Сделаем коэффициенты b и c нулями. Тогда уравнение примет вид:

Получили уравнение 2x= 0. Левая часть является произведением, а правая часть равна нулю. Произведение равно нулю, если хотя бы один из сомножителей равен нулю. Очевидно, что = 0. Действительно, 2 × 0= 0. Отсюда, 0 = 0. При других значениях x равенства достигаться не будет.

Проще говоря, если в квадратном уравнении вида axbx = 0 числа b и с равны нулю, то корень такого уравнения равен нулю.

Отметим, что когда употребляются словосочетания «b равно нулю» или «с равно нулю«, то подразумевается, что параметры b или c вовсе отсутствуют в уравнении.

Например, если дано уравнение 2x− 32 = 0, то мы говорим, что = 0. Потому что если сравнить с полным уравнением axbx = 0, то можно заметить, что в уравнении 2x− 32 = 0 присутствует старший коэффициент a, равный 2; присутствует свободный член −32; но отсутствует коэффициент b.

Наконец, рассмотрим полное квадратное уравнение axbx = 0. В качестве примера решим квадратное уравнение x− 2+ 1 = 0.

Итак, требуется найти x, при котором левая часть станет равна нулю. Воспользуемся изученными ранее тождественными преобразованиями.

Прежде всего заметим, что левая часть уравнения представляет собой квадрат разности двух выражений. Если мы вспомним как раскладывать многочлен на множители, то получим в левой части (− 1)2.

Рассуждаем дальше. Левая часть возведенá в квадрат и она равна нулю. Какое число в квадрате равно нулю? Очевидно, что только 0. Поэтому наша задача найти x, при котором выражение − 1 равно нулю. Решив простейшее уравнение − 1 = 0, можно узнать чему равно x

Этот же результат можно получить, если воспользоваться квадратным корнем. В уравнении (− 1)= 0 выражение (− 1) представляет собой квадратный корень из нуля. Тогда можно записать, что . В этом примере записывать перед корнем знак ± не нужно, поскольку корень из нуля имеет только одно значение — ноль. Тогда получается − 1 = 0. Отсюда = 1.

Значит корнем уравнения x− 2+ 1 = 0 является единица. Других корней у данного уравнения нет. В данном случае мы решили квадратное уравнение, имеющее только один корень. Такое тоже бывает.

Не всегда бывают даны простые уравнения. Рассмотрим например уравнение x+ 2− 3 = 0.

В данном случае левая часть уже не является квадратом суммы или разности. Поэтому нужно искать другие пути решения.

Заметим, что левая часть уравнения представляет собой квадратный трехчлен. Тогда можно попробовать выделить полный квадрат из этого трёхчлена и посмотреть что это нам даст.

Выделим полный квадрат из квадратного трёхчлена, располагающего в левой части уравнения:

В получившемся уравнении перенесем −4 в правую часть, изменив знак:

Теперь воспользуемся квадратным корнем. В уравнении (+ 1)= 4 выражение (+ 1) представляет собой квадратный корень из числа 4. Тогда можно записать, что . Вычисление правой части даст выражение + 1 = ±2. Отсюда полýчится два уравнения: + 1 = 2 и + 1 = −2, корнями которых являются числа 1 и −3

Значит корнями уравнения x+ 2− 3 = 0 являются числа 1 и −3.

Выполним проверку:


Пример 3. Решить уравнение x− 6+ 9 = 0, выделив полный квадрат.

Выделим полный квадрат из левой части:

Далее воспользуемся квадратным корнем и узнáем чему равно x

Значит корнем уравнения x− 6+ 9 = 0 является 3. Выполним проверку:


Пример 4. Решить квадратное уравнение 4x+ 28− 72 = 0, выделив полный квадрат:

Выделим полный квадрат из левой части:

Перенесём −121 из левой части в правую часть, изменив знак:

Воспользуемся квадратным корнем:

Получили два простых уравнения: 2+ 7 = 11 и 2+ 7 = −11. Решим их:


Пример 5. Решить уравнение 2x+ 3− 27 = 0

Это уравнение немного посложнее. Когда мы выделяем полный квадрат, первый член квадратного трёхчлена мы представляем в виде квадрата какого-нибудь выражения.

Так, в прошлом примере первым членом уравнения был 4x2. Его можно было представить в виде квадрата выражения 2x, то есть (2x)= 22x= 4x2. Чтобы убедиться что это правильно, можно извлечь квадратный корень из выражения 4x2. Это квадратный корень из произведения — он равен произведению корней:

В уравнении 2x+ 3− 27 = 0 первый член это 2x2. Его нельзя представить в виде квадрата какого-нибудь выражения. Потому что нет числá, квадрат которого равен 2. Если бы такое число было, то этим числом был бы квадратный корень из числа 2. Но квадратный корень из числа 2 извлекается только приближённо. А приближённое значение не годится для представления числá 2 в виде квадрата.

Если обе части исходного уравнения умножить или разделить на одно и то же число, то полýчится уравнение равносильное исходному. Это правило сохраняется и для квадратного уравнения.

Тогда можно разделить обе части нашего уравнения на 2. Это позвóлит избавиться от двойки перед x2 что впоследствии даст нам возможность выделить полный квадрат:

Перепишем левую часть в виде трёх дробей со знаменателем 2

Сократим первую дробь на 2. Остальные члены левой части перепишем без изменений. Правая часть по-прежнему станет равна нулю:

Выделим полный квадрат.

При представлении члена в виде удвоенного произведения, появление множителя 2 привело бы к тому, что этот множитель и знаменатель дроби сократились бы. Чтобы этого не произошло, удвоенное произведение было домножено на . При выделении полного квадрата всегда нужно стараться сделать так, чтобы значение изначального выражения не изменилось.

Свернём полученный полный квадрат:

Приведём подобные члены:

Перенесём дробь в правую часть, изменив знак:

Воспользуемся квадратным корнем. Выражение представляет собой квадратный корень из числа

Для вычисления правой части воспользуемся правилом извлечения квадратного корня из дроби:

Тогда наше уравнение примет вид:

Полýчим два уравнения:

Решим их:

Значит корнями уравнения 2x+ 3− 27 = 0 являются числа 3 и .

Корень удобнее оставить в таком виде, не выполняя деления числителя на знаменатель. Так проще будет выполнять проверку.

Выполним проверку. Подставим найденные корни в исходное уравнение:

В обоих случаях левая часть равна нулю, значит уравнение 2x+ 3− 27 = 0 решено верно.

Решая уравнение 2x+ 3− 27 = 0, в самом начале мы разделили обе его части на 2. В результате получили квадратное уравнение, в котором коэффициент перед x2 равен единице:

Такой вид квадратного уравнения называют приведённым квадратным уравнением.

Любое квадратное уравнение вида axbx = 0 можно сделать приведённым. Для этого нужно разделить обе его части на коэффициент, который располагается перед x². В данном случае обе части уравнения axbx = 0 нужно разделить на a


Пример 6. Решить квадратное уравнение 2x+ 2 = 0

Сделаем данное уравнение приведённым:

Выделим полный квадрат:

Получили уравнение , в котором квадрат выражения равен отрицательному числу . Такого быть не может, поскольку квадрат любого числа или выражения всегда положителен.

Следовательно, нет такого значения x, при котором левая часть стала бы равна . Значит уравнение не имеет корней.

А поскольку уравнение равносильно исходному уравнению 2x+ 2 = 0, то и оно (исходное уравнение) не имеет корней.


Формулы корней квадратного уравнения

Выделять полный квадрат для каждого решаемого квадратного уравнения не очень удобно.

Можно ли создать универсальные формулы для решения квадратных уравнений? Оказывается можно. Сейчас мы этим и займёмся.

Взяв за основу буквенное уравнение axbx = 0, и выполнив некоторые тождественные преобразования, мы сможем получить формулы для вывода корней квадратного уравнения axbx = 0. В эти формулы можно будет подставлять коэффициенты a, b, с и получать готовые решения.

Итак, выделим полный квадрат из левой части уравнения axbx = 0. Сначала сделаем данное уравнение приведённым. Разделим обе его части на a

Теперь в получившемся уравнении выделим полный квадрат:

Перенесем члены и в правую часть, изменив знак:

Приведём правую часть к общему знаменателю. Дроби, состоящие из букв, привóдят к общему знаменателю методом «крест-нáкрест». То есть знаменатель первой дроби станóвится дополнительным множителем второй дроби, а знаменатель второй дроби станóвится дополнительным множителем первой дроби:

В числителе правой части вынесем за скобки a

Сократим правую часть на a

Поскольку все преобразования были тождественными, то получившееся уравнение имеет те же корни, что и исходное уравнение axbx = 0.

Уравнение будет иметь корни только тогда, если правая часть больше нуля или равна нулю. Это потому что в левой части выполнено возведéние в квадрат, а квадрат любого числа положителен или равен нулю (если в этот квадрат возвóдится ноль). А чему будет равна правая часть зависит от того, что будет подставлено вместо переменных a, b и c.

Поскольку при любом a не рáвным нулю, знаменатель правой части уравнения всегда будет положительным, то знак дроби будет зависеть от знака её числителя, то есть от выражения b− 4ac.

Выражение b− 4ac называют дискриминантом квадратного уравнения. Дискриминант это латинское слово, означающее различитель. Дискриминант квадратного уравнения обозначается через букву D

D = b2 4ac

Дискриминант позволяет заранее узнать имеет ли уравнение корни или нет. Так, в предыдущем задании мы долго решали уравнение 2x+ 2 = 0 и оказалось, что оно не имеет корней. Дискриминант же позволил бы нам заранее узнать, что корней нет. В уравнении 2x+ 2 = 0 коэффициенты a, b и c равны 2, 1 и 2 соответственно. Подставим их в формулу D = b2−4ac

D = b2 − 4ac = 12 − 4 × 2 × 2 = 1 − 16 = −15.

Видим, что D (оно же b− 4ac) является отрицательным числом. Тогда нет смысла решать уравнение 2x+ 2 = 0, выделяя в нём полный квадрат, потому что когда мы дойдем до уравнения вида , окажется что правая часть станет меньше нуля (из-за отрицательного дискриминанта). А квадрат числа не может быть отрицательным. Следовательно, корней у данного уравнения не будет.

Станóвится понятно почему древние люди считали выражение b− 4ac различителем. Это выражение подобно индикатору позволяет различить уравнение имеющего корни от уравнения, не имеющего корней.

Итак, D равно b− 4ac. Подставим в уравнении вместо выражения b− 4ac букву D

Если дискриминант исходного уравнения окажется меньше нуля (< 0), то уравнение примет вид:

В этом случае говорят, что у исходного уравнения корней нет, поскольку квадрат любого числа не должен быть отрицательным.

Если дискриминант исходного уравнения окажется больше нуля (> 0), то уравнение примет вид:

В этом случае уравнение будет иметь два корня. Для их вывода воспользуемся квадратным корнем:

Получили уравнение . Из него полýчится два уравнения: и . Выразим x в каждом из уравнений:

Получившиеся два равенства это и есть универсальные формулы для решения квадратного уравнения axbx = 0. Их называют формулами корней квадратного уравнения.

Чаще всего эти формулы обозначаются как x1 и x2. То есть для вычисления первого корня используется формула c индексом 1; для вывода второго корня — формула с индексом 2. Обозначим свои формулы так же:

Очерёдность применения формул не важнá.

Решим например квадратное уравнение x+ 2− 8 = 0 с помощью формул корней квадратного уравнения. Коэффициенты данного квадратного уравнения это числа 1, 2 и −8. То есть, = 1, = 2, = −8.

Прежде чем использовать формулы корней квадратного уравнения, нужно найти дискриминант этого уравнения.

Найдём дискриминант квадратного уравнения. Для этого воспользуемся формулой D = b2 4ac. Вместо переменных a, b и c у нас будут коэффициенты уравнения x+ 2− 8 = 0

D = b2 4ac = 22− 4 × 1 × (−8) = 4 + 32 = 36

Дискриминант больше нуля. Значит уравнение имеет два корня. Теперь можно воспользоваться формулами корней квадратного уравнения:

Значит корнями уравнения x+ 2− 8 = 0 являются числа 2 и −4. Проверкой убеждаемся, что корни найдены верно:

Наконец, рассмотрим случай когда дискриминант квадратного уравнения равен нулю. Вернёмся к уравнению . Если дискриминант равен нулю, то правая часть уравнения примет вид:

И в этом случае квадратное уравнение будет иметь только один корень. Воспользуемся квадратным корнем:

Далее выражаем x

Это ещё одна формула для вывода корня квадратного корня. Рассмотрим её применение. Ранее мы решили уравнение x− 6+ 9 = 0, имеющее один корень 3. Решили мы его методом выделения полного квадрата. Теперь попробуем решить с помощью формул.

Найдём дискриминант квадратного уравнения. В этом уравнении = 1, = −6, = 9. Тогда по формуле дискриминанта имеем:

D = b2 4ac = (−6)− 4 × 1 × 9 = 36 − 36 = 0

Дискриминант равен нулю (= 0). Это означает, что уравнение имеет только один корень, и вычисляется он по формуле

Значит корнем уравнения x− 6+ 9 = 0 является число 3.

Для квадратного уравнения, имеющего один корень также применимы формулы и . Но применение каждой из них будет давать один и тот же результат.

Применим эти две формулы для предыдущего уравнения. В обоих случаях получим один и тот же ответ 3

Если квадратное уравнение имеет только один корень, то желательно применять формулу , а не формулы и . Это позволяет сэкономить время и место.


Пример 3. Решить уравнение 5x− 6+ 1 = 0

Найдём дискриминант квадратного уравнения:

Дискриминант больше нуля. Значит уравнение имеет два корня. Воспользуемся формулами корней квадратного уравнения:

Значит корнями уравнения 5x− 6+ 1 = 0 являются числа 1 и .

Ответ: 1; .


Пример 4. Решить уравнение x+ 4+ 4 = 0

Найдём дискриминант квадратного уравнения:

Дискриминант равен нулю. Значит уравнение имеет только один корень. Он вычисляется по формуле

Значит корнем уравнения x+ 4+ 4 = 0 является число −2.

Ответ: −2.


Пример 5. Решить уравнение 3x+ 2+ 4 = 0

Найдём дискриминант квадратного уравнения:

Дискриминант меньше нуля. Значит корней у данного уравнения нет.

Ответ: корней нет.


Пример 6. Решить уравнение (+ 4)= 3+ 40

Приведём данное уравнение к нормальному виду. В левой части располагается квадрата суммы двух выражений. Раскрóем его:

Перенесём все члены из правой части в левую часть, изменив их знаки. В правой части останется ноль:

Приведём подобные члены в левой части:

В получившемся уравнении найдём дискриминант:

Дискриминант больше нуля. Значит уравнение имеет два корня. Воспользуемся формулами корней квадратного уравнения:

Значит корнями уравнения (+ 4)= 3+ 40 являются числа 3 и −8.

Ответ: 3; −8.


Пример 7. Решить уравнение

Умнóжим обе части данного уравнения на 2. Это позвóлит нам избавиться от дроби в левой части:

В получившемся уравнении перенесём 22 из правой части в левую часть, изменив знак. В правой части останется 0

Приведём подобные члены в левой части:

В получившемся уравнении найдём дискриминант:

Дискриминант больше нуля. Значит уравнение имеет два корня. Воспользуемся формулами корней квадратного уравнения:

Значит корнями уравнения являются числа 23 и −1.

Ответ: 23; −1.


Пример 8. Решить уравнение

Умнóжим обе части на наименьшее общее кратное знаменателей обеих дробей. Это позвóлит избавиться от дробей в обеих частях. Наименьшее общее кратное чисел 2 и 3 это число 6. Тогда получим:

В получившемся уравнении раскроем скобки в обеих частях:

Теперь перенесём все члены из правой части в левую часть, изменив у них знаки. В правой части останется 0

Приведём подобные члены в левой части:

В получившемся уравнении найдём дискриминант:

Дискриминант больше нуля. Значит уравнение имеет два корня. Воспользуемся формулами корней квадратного уравнения:

Значит корнями уравнения являются числа и 2.


Примеры решения квадратных уравнений

Пример 1. Решить уравнение x= 81

Это простейшее квадратное уравнение, в котором надо определить число, квадрат которого равен 81. Таковыми являются числа 9 и −9. Воспользуемся квадратным корнем для их вывода:

Ответ: 9, −9.


Пример 2. Решить уравнение x− 9 = 0

Это неполное квадратное уравнение. Для его решения нужно перенести член −9 в правую часть, изменив знак. Тогда получим:

Ответ: 3, −3.


Пример 3. Решить уравнение x− 9= 0

Это неполное квадратное уравнение. Для его решения сначала нужно вынести x за скобки:

Левая часть уравнения является произведением. Произведение равно нулю, если хотя один из сомножителей равен нулю.

Левая часть станет равна нулю, если отдельно x равно нулю, или если выражение − 9 равно нулю. Получится два уравнения, одно из которых уже решено:

Ответ: 0, 9.


Пример 4. Решить уравнение x+ 4− 5 = 0

Это полное квадратное уравнение. Его можно решить методом выделения полного квадрата или с помощью формул корней квадратного уравнения.

Решим данное уравнение с помощью формул. Сначала найдём дискриминант:

D = b− 4ac = 4− 4 × 1 × (−5) = 16 + 20 = 36

Дискриминант больше нуля. Значит уравнение имеет два корня. Вычислим их:

Ответ: 1, −5.


Пример 5. Решить уравнение

Умнóжим обе части на наименьшее общее кратное чисел 5, 3 и 6. Это позвóлит избавиться от дробей в обеих частях:

В получившемся уравнении перенесём все члены из правой части в левую часть, изменив знак. В правой части останется ноль:

Приведём подобные члены:

Решим получившееся уравнение с помощью формул:

Ответ: 5, .


Пример 6. Решить уравнение x= 6

В данном примере как и в первом нужно воспользоваться квадратным корнем:

Однако, квадратный корень из числа 6 не извлекается. Он извлекается только приближённо. Корень можно извлечь с определённой точностью. Извлечём его с точностью до сотых:

Но чаще всего корень оставляют в виде радикала:

Ответ:


Пример 7. Решить уравнение (2+ 3)+ (− 2)= 13

Раскроем скобки в левой части уравнения:

В получившемся уравнении перенесём 13 из правой части в левую часть, изменив знак. Затем приведём подобные члены:

Получили неполное квадратное уравнение. Решим его:

Ответ: 0, −1,6.


Пример 8. Решить уравнение (5 + 7x)(4 − 3x) = 0

Данное уравнение можно решить двумя способами. Рассмотрим каждый из них.

Первый способ. Раскрыть скобки и получить нормальный вид квадратного уравнения.

Раскроем скобки:

Приведём подобные члены:

Перепишем получившееся уравнение так, чтобы член со старшим коэффициентом располагался первым, член со вторым коэффициентом — вторым, а свободный член располагался третьим:

Чтобы старший член стал положительным, умнóжим обе части уравнения на −1. Тогда все члены уравнения поменяют свои знаки на противоположные:

Решим получившееся уравнение с помощью формул корней квадратного уравнения:

Второй способ. Найти значения x, при которых сомножители левой части уравнения равны нулю. Этот способ удобнее и намного короче.

Произведение равно нулю, если хотя бы один из сомножителей равен нулю. В данном случае равенство в уравнении (5 + 7x)(4 − 3x) = 0 будет достигаться, если выражение (5 + 7x) равно нулю, или же выражение (4 − 3x) равно нулю. Наша задача выяснить при каких x это происходит:


Примеры решения задач

Предстáвим, что возникла необходимость построить небольшую комнату, площадь которой 8 м2. При этом длина комнаты должна быть в два раза больше её ширины. Как определить длину и ширину такой комнаты?

Сделаем примерный рисунок этой комнаты, который иллюстрирует вид сверху:

Обозначим ширину комнаты через x. А длину комнаты через 2x, потому что по условию задачи длина должна быть в два раза больше ширины. Множитель 2 и выполнит это требование:

Поверхность комнаты (её пол) является прямоугольником. Для вычисления площади прямоугольника, нужно длину данного прямоугольника умножить на его ширину. Сделаем это:

2x × x

По условию задачи площадь должна быть 8 м2. Значит выражение 2× x следует приравнять к 8

2x × x = 8

Получилось уравнение. Если решить его, то можно найти длину и ширину комнаты.

Первое что можно сделать это выполнить умножение в левой части уравнения:

2x2 = 8

В результате этого преобразования переменная x перешла во вторую степень. А мы говорили, что если переменная, входящая в уравнение, возведенá во вторую степень (в квадрат), то такое уравнение является уравнением второй степени или квадратным уравнением.

Для решения нашего квадратного уравнения воспользуемся изученными ранее тождественными преобразованиями. В данном случае можно разделить обе части на 2

Теперь воспользуемся квадратным корнем. Если x= 4, то . Отсюда = 2 и = −2.

Через x была обозначена ширина комнаты. Ширина не должна быть отрицательной, поэтому в расчёт берём только значение 2. Такое часто бывает при решении задачи, в которых применяется квадратное уравнение. В ответе получаются два корня, но условию задачи удовлетворяет только один из них.

А длина была обозначена через 2x. Значение x теперь известно, подставим его в выражение 2x и вычислим длину:

2x = 2 × 2 = 4

Значит длина равна 4 м, а ширина 2 м. Это решение удовлетворяет условию задачи, поскольку площадь комнаты равна 8 м2

4 × 2 = 8 м2

Ответ: длина комнаты составляет 4 м, а ширина 2 м.


Пример 2. Огородный участок, имеющий форму прямоугольника, одна сторона которого на 10 м больше другой, требуется обнести изгородью. Определить длину изгороди, если известно, что площадь участка равна 1200 м2

Решение

Длина прямоугольника, как правило, больше его ширины. Пусть ширина участка x метров, а длина (+ 10) метров. Площадь участка составляет 1200 м2. Умножим длину участка на его ширину и приравняем к 1200, получим уравнение:

x(x + 10) = 1200

Решим данное уравнение. Для начала раскроем скобки в левой части:

Перенесём 1200 из правой части в левую часть, изменив знак. В правой части останется 0

Решим получившееся уравнение с помощью формул:

Несмотря на то, что квадратное уравнение имеет два корня, в расчёт берём только значение 30. Потому что ширина не может выражаться отрицательным числом.

Итак, через x была обозначена ширина участка. Она равна тридцати метрам. А длина была обозначена через выражение + 10. Подставим в него найденное значение x и вычислим длину:

x + 10 = 30 + 10 = 40 м

Значит длина участка составляет сорок метров, а ширина тридцать метров. Эти значения удовлетворяют условию задачи, поскольку если перемножить длину и ширину (числа 40 и 30) получится 1200 м2

40 × 30 = 1200 м2

Теперь ответим на вопрос задачи. Какова длина изгороди? Чтобы её вычислить нужно найти периметр участка.

Периметр прямоугольника это сумма всех его сторон. Тогда:

P = 2(a + b) = 2 × (40 + 30) = 2 × 70 = 140 м.

Ответ: длина изгороди огородного участка составляет 140 м.


Задания для самостоятельного решения

Задание 1. Решить уравнение:

Решение:

Ответ: 2; −2.

Задание 2. Решить уравнение:

Решение:

Ответ: корней нет.

Задание 3. Решить уравнение:

Решение:

Ответ: 3; −3.

Задание 4. Решить уравнение, используя выделение полного квадрата:

Решение:

Ответ: 3; −13.

Задание 5. Решить уравнение, используя выделение полного квадрата:

Решение:

Ответ: 12; 4.

Задание 6. Решить уравнение, используя выделение полного квадрата:

Решение:

Ответ: 7; 5.

Задание 7. Решить уравнение:

Решение:

Ответ: 0; 1.

Задание 8. Решить уравнение:

Решение:

Ответ: 0; −3.

Задание 9. Решить уравнение:

Решение:

Ответ: 7; −7.

Задание 10. Решить уравнение:

Решение:

Ответ:

Задание 11. Решить уравнение:

Решение:

Ответ: 5; −5.

Задание 12. Решить уравнение:

Решение:

Ответ: 7; 2

Задание 13. Решить уравнение:

Решение:

Ответ: корней нет.

Задание 14. Решить уравнение:

Решение:

Ответ:

Задание 15. Решить уравнение:

Решение:

Ответ: 1; −5.

Задание 16. Решить уравнение:

Решение:

Ответ: 5; −9.

Задание 17. Решить уравнение:

Решение:

Ответ: −3; −4.

Задание 18. Решить уравнение:

Решение:

Ответ: .

Понравился урок?
Вступай в нашу новую группу Вконтакте и начни получать уведомления о новых уроках

Возникло желание поддержать проект?
Используй кнопку ниже

Навигация по записям

Решение квадратных уравнений онлайн

С помощю этого онлайн калькулятора можно найти решение (корни) квадратного уравнения. Дается подробное решение с пояснениями. Для нахождения решений квадратного уравнения введите коэффициенты уравнения и нажмите на кнопку «Решить». Теоретическую часть и численные примеры смотрите ниже.

Очистить все ячейки?

Инструкция ввода данных. Числа вводятся в виде целых чисел (примеры: 487, 5, -7623 и т.д.), десятичных чисел (напр. 67., 102.54 и т.д.) или дробей. Дробь нужно набирать в виде a/b, где a и b (b>0) целые или десятичные числа. Примеры 45/5, 6.6/76.4, -7/6.7 и т.д.

 

Корни квадратного уравнения − теория примеры и решения

Квадратным уравнением называется уравнение следующего вида:

где x−переменная, а a, b, c некоторые числа (a≠0). Числа a, b, c называются коэффициентами квадратного уравнения. Коэффицинт c называется свободным членом.

Если a=1, то квадратное уравнение называется приведенным. Заметим, что любое квадратное уравнение можно привести к приведенному виду, разделив обе части уравнения на a. Действительно:

Если в квадратном уравнении (1) один из коэффициентов b, c равен нулю или оба коэффициента b, c равны нулю, то квадратное уравнение называется неполным.

Рассмотрим разные виды неполных квадратных уравнений.

1. При b=0 имеем:

Для решения этого уравнения свободный член перенесем в правую часть уравнения:

Решая последнее уравнение относительно x получим корни квадратного уравнения (3):

Если , то квадратное уравнение не имеет действительных корней.

2. При c=0 имеем:

Разложим левую часть последнего уравнения на множители:

Из (4) следует x=0 или ax+b=0. Следовательно имеем следующие решения:

3. При b=0, c=0 имеем:

и, следовательно

Рассмотрим, далее, алгоритм решения квадратных уравнений общего вида (1). Разделим обе части уравнения на a:

Сделаем эквивалентные преобразования уравнения (5):

Легко догадаться, что первые три слагаемые уравнения (6) образуют квадрат следующей суммы:

Тогда

Обозначим

D− называется дискриминантом квадратного уравнения (1). Так как a≠0, то 4a2>0. Знак правой части уравнения (7) определяется знаком дискриминанта D.

Учитывая (8) запишем (7) в следующем виде:

При решении последнего уравнения возможны следующие варианты:

1. При D>0, имеем

Таким образом, при D>0, квадратное уравнение (1) имеет две корни:

2.При D=0, имеем

То есть, при D=0 квадратное уравнение (1) имеет единственный корень:

3. При D<0, правая часть уравнения (9) отрицательна, а так как квадрат числа не может быть отрицательным числом, то квадратное уравнение (1) не имеет корней.

Пример 1. Решить квадратное уравнение

.(10)

Решение. Запишем коэффициенты квадратного уравнения (10):

Вычислим дискриминант квадратного уравнения:

.

Дискриминант положительное число. Следовательно квадратное уравнение (10) имеет два решения.

Найдем решение квадратного уравнения используя следующую формулу:

.(11)

Подставляя значения коэффициентов a, b, c, D в (11), получим:

,
.

Ответ:

Пример 2. Решить следующее квадратное уравнение:

.(12)

Решение. Запишем коэффициенты квадратного уравнения (12):

Вычислим дискриминант квадратного уравнения:

.

Дискриминант равен нулю. Следовательно квадратное уравнение (12) имеет единственное решение. Найдем решение квадратного уравнения используя следующую формулу:

.(13)

Подставляя значения коэффициентов a, b, c, D в (13), получим:

,

Ответ:

.

Пример 3. Решить следующее квадратное уравнение:

.(14)

Решение. Запишем коэффициенты квадратного уравнения (14):

Вычислим дискриминант квадратного уравнения:

.

Дискриминант отрицательное число. Следовательно квадратное уравнение (14) не имеет действительных корней.

Ответ: Квадратное уравнение не имеет действительных корней.

Решить квадратное уравнение онлайн. Решение уравнений с параметром

Цели:

  1. Систематизировать и обобщить знания и умения по теме: Решения уравнений третьей и четвертой степени.
  2. Углубить знания, выполнив ряд заданий, часть из которых не знакома или по своему типу, или способу решения.
  3. Формирование интереса к математике через изучение новых глав математики, воспитание графической культуры через построение графиков уравнений.

Тип урока : комбинированный.

Оборудование: графопроектор.

Наглядность: таблица «Теорема Виета».

Ход урока

1. Устный счет

а) Чему равен остаток от деления многочлена р n (х) = а n х n + а n-1 х n-1 + … + а 1 х 1 + a 0 на двучлен х-а?

б) Сколько корней может иметь кубическое уравнение?

в) С помощью чего мы решаем уравнение третьей и четвертой степени?

г) Если b четное число в квадратном уравнение, то чему равен Д и х 1 ;х 2

2. Самостоятельная работа (в группах)

Составить уравнение, если известны корни (ответы к заданиям закодированы) Используется «Теорема Виета»

1 группа

Корни: х 1 = 1; х 2 = -2; х 3 = -3; х 4 = 6

Составить уравнение:

B=1 -2-3+6=2; b=-2

с=-2-3+6+6-12-18= -23; с= -23

d=6-12+36-18=12; d= -12

е=1(-2)(-3)6=36

х 4 — 2 х 3 — 23х 2 — 12 х + 36 = 0 (это уравнение решает потом 2 группа на доске)

Решение . Целые корни ищем среди делителей числа 36.

р = ±1;±2;±3;±4;±6…

р 4 (1)=1-2-23-12+36=0 Число 1 удовлетворяет уравнению, следовательно, =1 корень уравнения. По схеме Горнера

р 3 (x) = х 3 -х 2 -24x -36

р 3 (-2) = -8 -4 +48 -36=0, х 2 =-2

р 2 (x) = х 2 -3х -18=0

х 3 =-3, х 4 =6

Ответ: 1;-2;-3;6 сумма корней 2 (П)

2 группа

Корни: х 1 = -1; х 2 = х 3 =2; х 4 =5

Составить уравнение:

B=-1+2+2+5-8; b= -8

с=2(-1)+4+10-2-5+10=15; с=15

D=-4-10+20-10= -4; d=4

е=2(-1)2*5=-20;е=-20

8+15+4х-20=0 (это уравнение решает на доске 3 группа)

р = ±1;±2;±4;±5;±10;±20.

р 4 (1)=1-8+15+4-20=-8

р 4 (-1)=1+8+15-4-20=0

р 3 (x) = х 3 -9х 2 +24x -20

р 3 (2) = 8 -36+48 -20=0

р 2 (x) = х 2 -7х +10=0 х 1 =2; х 2 =5

Ответ: -1;2;2;5 сумма корней 8(Р)

3 группа

Корни: х 1 = -1; х 2 =1; х 3 =-2; х 4 =3

Составить уравнение:

В=-1+1-2+3=1;в=-1

с=-1+2-3-2+3-6=-7;с=-7

D=2+6-3-6=-1; d=1

е=-1*1*(-2)*3=6

х 4 — х 3 — 7х 2 + х + 6 = 0 (это уравнение решает потом на доске 4 группа)

Решение. Целые корни ищем среди делителей числа 6.

р = ±1;±2;±3;±6

р 4 (1)=1-1-7+1+6=0

р 3 (x) = х 3 — 7x -6

р 3 (-1) = -1+7-6=0

р 2 (x) = х 2 -х -6=0; х 1 =-2; х 2 =3

Ответ:-1;1;-2;3 Сумма корней 1(О)

4 группа

Корни: х 1 = -2; х 2 =-2; х 3 =-3; х 4 =-3

Составить уравнение:

B=-2-2-3+3=-4; b=4

с=4+6-6+6-6-9=-5; с=-5

D=-12+12+18+18=36; d=-36

е=-2*(-2)*(-3)*3=-36;е=-36

х 4 + 4х 3 – 5х 2 – 36х -36 = 0 (это уравнение решает потом 5 группа на доске)

Решение. Целые корни ищем среди делителей числа -36

р = ±1;±2;±3…

р(1)= 1 + 4-5-36-36 = -72

р 4 (-2) = 16 -32 -20 + 72 -36 = 0

р 3 (х) = х 3 +2х 2 -9х-18 = 0

р 3 (-2)= -8 + 8 + 18-18 = 0

р 2 (х) = х 2 -9 = 0; x=±3

Ответ: -2; -2; -3; 3 Сумма корней-4 (Ф)

5 группа

Корни: х 1 = -1; х 2 =-2; х 3 =-3; х 4 =-4

Составить уравнение

х 4 + 10х 3 + 35х 2 + 50х + 24 = 0 (это уравнение решает потом 6группа на доске)

Решение . Целые корни ищем среди делителей числа 24.

р = ±1;±2;±3

р 4 (-1) = 1 -10 + 35 -50 + 24 = 0

р 3 (х) = x- 3 + 9х 2 + 26x+ 24 = 0

p 3 (-2) = -8 + 36-52 + 24 = О

р 2 (х) = x 2 + 7x+ 12 = 0

Ответ:-1;-2;-3;-4 сумма-10 (И)

6 группа

Корни: х 1 = 1; х 2 = 1; х 3 = -3; х 4 = 8

Составить уравнение

B=1+1-3+8=7;b=-7

с=1 -3+8-3+8-24= -13

D=-3-24+8-24= -43; d=43

х 4 — 7х 3 — 13х 2 + 43 x — 24 = 0 (это уравнение решает потом 1 группа на доске)

Решение . Целые корни ищем среди делителей числа -24.

р 4 (1)=1-7-13+43-24=0

р 3 (1)=1-6-19+24=0

р 2 (x)= х 2 -5x — 24 = 0

х 3 =-3, х 4 =8

Ответ: 1;1;-3;8 сумма 7 (Л)

3. Решение уравнений с параметром

1. Решить уравнение х 3 + 3х 2 + mх — 15 = 0; если один из корней равен (-1)

Ответ записать в порядке возрастания

R=Р 3 (-1)=-1+3-m-15=0

х 3 + 3х 2 -13х — 15 = 0; -1+3+13-15=0

По условию х 1 = — 1; Д=1+15=16

Р 2 (х) = х 2 +2х-15 = 0

х 2 =-1-4 = -5;

х 3 =-1 + 4 = 3;

Ответ:- 1;-5; 3

В порядке возрастания: -5;-1;3. (Ь Н Ы)

2. Найти все корни многочлена х 3 — 3х 2 + ах — 2а + 6, если остатки от его деления на двучлены х-1 и х +2 равны.

Решение: R=Р 3 (1) = Р 3 (-2)

Р 3 (1) = 1-3 + а- 2а + 6 = 4-а

Р 3 (-2) = -8-12-2а-2а + 6 = -14-4а

x 3 -Зх 2 -6х + 12 + 6 = х 3 -Зх 2 -6х + 18

x 2 (x-3)-6(x-3) = 0

(х-3)(х 2 -6) = 0

3) а=0, х 2 -0*х 2 +0 = 0; х 2 =0; х 4 =0

а=0; х=0; х=1

а>0; х=1; х=а ± √а

2. Составить уравнение

1 группа . Корни: -4; -2; 1; 7;

2 группа . Корни: -3; -2; 1; 2;

3 группа . Корни: -1; 2; 6; 10;

4 группа . Корни: -3; 2; 2; 5;

5 группа . Корни: -5; -2; 2; 4;

6 группа . Корни: -8; -2; 6; 7.

Представление об уравнениях с двумя переменными впервые формируется в курсе математики за 7 класс. Рассматриваются конкретные задачи, процесс решения которых приводит к такому виду уравнений.

При этом они изучаются довольно поверхностно. В программе главный акцент делается на системах уравнений с двумя неизвестными.

Это стало причиной того, что задачи, в которых на коэффициенты уравнения накладываются определенные ограничения, практически не рассматриваются. Недостаточно внимания уделено методам решения заданий типа «Решить уравнение в натуральных или целых числах». Известно, что материалы ЕГЭ и билеты вступительных экзаменов часто содержат такие упражнения.

Какие именно уравнения определяются как уравнения с двумя переменными?

ху = 8, 7х + 3у = 13 или х 2 + у = 7 – примеры уравнений с двумя переменными.

Рассмотрим уравнение х – 4у = 16. Если х = 4, а у = -3, оно будет правильным равенством. Значит, эта пара значений – решение данного уравнения.

Решение любого уравнения с двумя переменными – множество пар чисел (х; у), которые удовлетворяют это уравнение (превращают его в верное равенство).

Часто уравнение преобразовывают так, чтобы из него можно было получить систему для нахождения неизвестных.

Примеры

Решить уравнение: ху – 4 = 4х – у.

В данном примере можно воспользоваться методом разложения на множители. Для этого нужно сгруппировать слагаемые и вынести общий множитель за скобки:

ху – 4 = 4х – у;

ху – 4 – 4х + у = 0;

(ху + у) – (4х + 4) = 0;

у(х + 1) – 4(х + 1) = 0;

(х + 1)(у — 4) = 0.

Ответ: Все пары (х; 4), где х – любое рациональное число и (-1; у), где у – любое рациональное число.

Решить уравнение: 4х 2 + у 2 + 2 = 2(2х — у).

Первый шаг – группирование.

4х 2 + у 2 + 2 = 4х – 2у;

4х 2 + у 2 + 1 — 4х + 2у + 1 = 0;

(4х 2 – 4х +1) + (у 2 + 2у + 1) = 0.

Применив формулу квадрата разности, получим:

(2х — 1) 2 + (у + 1) 2 = 0.

При суммировании двух неотрицательных выражений ноль получится только в том случае, если 2х – 1 = 0 и у + 1 = 0. Отсюда следует: х = ½ и у = -1.

Ответ: (1/2; -1).

Решить уравнение (х 2 – 6х + 10)(у 2 + 10у + 29) = 4.

Рационально применить оценочный метод, выделив полные квадраты в скобках.

((х — 3) 2 + 1)((у + 5) 2 + 4) = 4.

При этом (х — 3) 2 + 1 ≥ 1, а (у + 5) 2 + 4 ≥ 4. Тогда левая часть уравнения всегда не меньше 4. Равенство возможно в случае

(х — 3) 2 + 1 = 1 и (у + 5) 2 + 4 = 4. Следовательно, х = 3, у = -5.

Ответ: (3; -5).

Решить уравнение в целых числах: х 2 + 10у 2 = 15х + 3.

Можно записать это уравнение в таком виде:

х 2 = -10у 2 + 15х + 3. Если правую часть равенства делить на 5, то 3 – остаток. Из этого следует, что х 2 не делится на 5. Известно, что квадрат числа, которое не делится на 5, должен дать в остатке или 1, или 4. Значит, уравнение корней не имеет.

Ответ: Решений нет.

Не стоит расстраиваться из-за трудностей в поиске верного решения для уравнения с двумя переменными. Упорство и практика обязательно принесут свои плоды.

В этой статье мы будем учиться решать биквадратные уравнения.

Итак, уравнения какого вида называются биквадратными?
Все уравнения вида ах 4 + bx 2 + c = 0 , гдеа ≠ 0 , являющиеся квадратными относительно х 2 , и называются биквадратными уравнениями. Как видите, эта запись очень похожа на запись квадратного уравнения, поэтому и решать биквадратные уравнения будем используя формулы, которые мы применяли при решении квадратного уравнения.

Только нам необходимо будет ввести новую переменную, то есть обозначим х 2 другой переменной, например, у или t (или же любой другой буквой латинского алфавита).

Например, решим уравнение х 4 + 4х 2 ‒ 5 = 0.

Обозначим х 2 через у (х 2 = у ) и получим уравнение у 2 + 4у – 5 = 0.
Как видите, такие уравнения вы уже умеете решать.

Решаем полученное уравнение:

D = 4 2 – 4 (‒ 5) = 16 + 20 = 36, √D = √36 = 6.

у 1 = (‒ 4 – 6)/2= ‒ 10 /2 = ‒ 5,

у 2 = (‒ 4 + 6)/2= 2 /2 = 1.

Вернемся к нашей переменной х.

Получили, что х 2 = ‒ 5 и х 2 = 1.

Замечаем, что первое уравнение решений не имеет, а второе дает два решения: х 1 = 1 и х 2 = ‒1. Будьте внимательны, не потеряйте отрицательный корень (чаще всего получают ответ х = 1, а это не правильно).

Ответ: — 1 и 1.

Для лучшего усвоения темы разберем несколько примеров.

Пример 1. Решите уравнение 2х 4 ‒ 5 х 2 + 3 = 0.

Пусть х 2 = у, тогда 2у 2 ‒ 5у + 3 =0.

D = (‒ 5) 2 – 4· 2 · 3 = 25 ‒ 24 = 1, √D = √1 = 1.

у 1 = (5 – 1)/(2· 2) = 4 /4 =1, у 2 = (5 + 1)/(2· 2) = 6 /4 =1,5.

Тогда х 2 = 1 и х 2 = 1,5.

Получаем х 1 = ‒1, х 2 = 1, х 3 = ‒ √1,5 , х 4 = √1,5.

Ответ: ‒1; 1; ‒ √1,5; √1,5.

Пример 2. Решите уравнение 2х 4 + 5 х 2 + 2 = 0.

2у 2 + 5у + 2 =0.

D = 5 2 – 4 · 2 · 2 = 25 ‒ 16 = 9, √D = √9 = 3.

у 1 = (‒ 5 – 3)/(2 · 2) = ‒ 8 /4 = ‒2, у 2 = (‒5 + 3)/(2 · 2) = ‒ 2 /4 = ‒ 0,5.

Тогда х 2 = ‒ 2 и х 2 = ‒ 0,5. Обратите внимание, ни одно из этих уравнений не имеет решения.

Ответ: решений нет.

Неполные биквадратные уравнения — это когда b = 0 (ах 4 + c = 0) или же c = 0

(ах 4 + bx 2 = 0) решают как и неполные квадратные уравнения.


Пример 3. Решить уравнение х 4 ‒ 25х 2 = 0

Разложим на множители, вынесем х 2 за скобки и тогда х 2 (х 2 ‒ 25) = 0.

Получим х 2 = 0 или х 2 ‒ 25 = 0, х 2 = 25.

Тогда имеем корни 0; 5 и – 5.

Ответ: 0; 5; – 5.

Пример 4. Решить уравнение 5х 4 ‒ 45 = 0 .

х 2 = ‒ √9 (решений не имеет)

х 2 = √9, х 1 = ‒ 3, х 2 = 3.

Как видите, умея решать квадратные уравнения, вы сможете справиться и с биквадратными.

Если же у вас остались вопросы, записывайтесь на мои уроки. Репетиор Валентина Галиневская.

сайт, при полном или частичном копировании материала ссылка на первоисточник обязательна.

Предлагаем вам удобный бесплатный онлайн калькулятор для решения квадратных уравнений. Вы сможете быстро получить и разобраться, как они решаются, на понятных примерах.
Чтобы произвести решение квадратного уравнения онлайн , вначале приведите уравнение к общему виду:
ax 2 + bx + c = 0
Заполните соответственно поля формы:

Как решить квадратное уравнение

Как решить квадратное уравнение: Виды корней:
1. Привести квадратное уравнение к общему виду:
Общий вид Аx 2 +Bx+C=0
Пример: 3х — 2х 2 +1=-1 Приводим к -2х 2 +3х+2=0

2. Находим дискриминант D.
D=B 2 -4*A*C .
Для нашего примера D= 9-(4*(-2)*2)=9+16=25.

3. Находим корни уравнения.
x1=(-В+D 1/2)/2А.
Для нашего случая x1=(-3+5)/(-4)=-0,5
x2=(-В-D 1/2)/2А.
Для нашего примера x2=(-3-5)/(-4)=2
Если В — четное число, то дискриманант и корни удобнее считать по формулам:
D=К 2 -ac
x1=(-K+D 1/2)/А
x2=(-K-D 1/2)/А,
Где K=B/2

1. Действительные корни. Причем. x1 не равно x2
Ситуация возникает, когда D>0 и A не равно 0.

2. Действительные корни совпадают. x1 равно x2
Ситуация возникает, когда D=0. Однако при этом, ни А, ни В, ни С не должны быть равны 0.

3. Два комплексных корня. x1=d+ei, x2=d-ei, где i=-(1) 1/2
Ситуация возникает, когда D
4. Уравнение имеет одно решение.
A=0, B и C нулю не равны. Уравнение становиться линейным.

5. Уравнение имеет бесчисленное множество решений.
A=0, B=0, C=0.

6. Уравнение решений не имеет.
A=0, B=0, C не равно 0.


Для закрепления алгоритма, вот еще несколько показательных примеров решений квадратных уравнений .

Пример 1. Решение обычного квадратного уравнения с разными действительными корнями.
x 2 + 3x -10 = 0
В этом уравнении
А=1, B = 3, С=-10
D=B 2 -4*A*C = 9-4*1*(-10) = 9+40 = 49
квадратный корень будем обозначать, как число 1/2 !
x1=(-В+D 1/2)/2А = (-3+7)/2 = 2
x2=(-В-D 1/2)/2А = (-3-7)/2 = -5

Для проверки подставим:
(x-2)*(x+5) = x2 -2x +5x – 10 = x2 + 3x -10

Пример 2. Решение квадратного уравнения с совпадением действительных корней.
х 2 – 8x + 16 = 0
А=1, B = -8, С=16
D = k 2 – AC = 16 – 16 = 0
X = -k/A = 4

Подставим
(x-4)*(x-4) = (x-4)2 = X 2 – 8x + 16

Пример 3. Решение квадратного уравнения с комплексными корнями.
13х 2 – 4x + 1 = 0
А=1, B = -4, С=9
D = b 2 – 4AC = 16 – 4*13*1 = 16 — 52 = -36
Дискриминант отрицательный – корни комплексные.2+20=0.$

Ответ: $z_{1,2}=\pm 2i$ $z_{3,4}=\pm\sqrt 5i.$ 

 

Решение квадратных уравнений через производные / Хабр

Здравствуйте, уважаемые читатели. После прочтения статьи у вас, вероятно, возникнет закономерный вопрос: «А зачем, собственно, это надо?». В силу этого сперва считаю необходимым заблаговременно сообщить, что искомый метод решения квадратных уравнений представлен скорее с морально-эстетической стороны математики, нежели со стороны практического сухого применения. Также заранее извиняюсь перед теми читателями, которые посчитают мои дилетантские изречения неприемлемыми. Итак, начнем забивать гвозди микроскопом.

Имеем алгебраическое уравнение второй степени (оно же квадратное) в общем виде:

Перейдем от квадратного уравнения к квадратичной функции:

Где, очевидно, необходимо найти такие значения аргумента функции, в которых оная возвратила бы ноль.

Кажется, нужно просто решить квадратное уравнение с помощью теоремы Виета или через дискриминант. Но мы ведь собрались здесь не для этого. Давайте-ка лучше возьмем производную!

Исходя из определения физического смысла производной первого порядка ясно, что подставляя аргумент в получившуюся выше функцию мы (в частности) получим скорость изменения функции в заданной этим аргументом точке.

Что же дальше делать? Непонятно. А в любом непонятном случае нужно брать производную ещё раз:

На этот раз мы получили «скорость скорости» изменения функции (то бишь ускорение) в конкретной точке. Немного проанализировав полученное, можно сделать вывод, что «ускорением» является константа, которая не зависит от аргумента функции — запомним это.

Сейчас вспомним немного физику и равноускоренное движение (РУД). Что у нас есть в арсенале? Верно, имеется формула для определения координаты перемещения по оси при искомом движении:

Где — время, — начальная скорость, — ускорение.
Нетрудно заметить, что наша изначальная функция как раз представляет из себя РУД.

Разве формула перемещения для РУД не является следствием решения квадратного уравнения?Нет. Формула для РУД выше по факту есть результат взятия интеграла от формулы скорости при ПРУД. Или из графика можно найти площадь фигуры. Там вылезет трапеция.
Формула перемещения при РУД не вытекает из решения каких-либо квадратных уравнений. Это очень важно, иначе не было бы смысла статьи.

Теперь осталось разобраться что есть что, и чего нам не хватает.

«Ускорение» у нас уже есть — им является производная второго порядка , выведенная выше. А вот чтобы получить начальную скорость , нам нужно взять в общем-то любой (обозначим его как ) и подставить его в производную теперь уже первого порядка — ибо она и будет искомым.

В таком случае возникает вопрос, какой же нужно взять? Очевидно, такой, чтобы начальная скорость была равна нулю, чтобы формула «перемещения при РУД» стала иметь вид:

В таком случае составим уравнение для поиска :

[подставили в производную первого порядка ]

Корнем такого уравнения относительно будет:

А значением исходной функции при таком аргументе будет:

Вспомним, какой целью мы задались в самом начале: «необходимо найти такие значения аргумента функции, в которых оная возвратила бы ноль». Иными словами, нам от положения необходимо «дойти до нуля».

Так как теперь нам известна начальная скорость, ускорение и какой путь необходимо пройти, то настало время отметить следующее:

, также как и

Тогда, подставив все известные величины, получим:

Поделим все на :

Теперь становится очевидно, что:

Соединим все «детали пазла» воедино:

Вот мы и получили окончательное решение поставленной задачи. Вообще Америку мы не открыли — мы просто пришли к формуле решения квадратного уравнения через дискриминант окольными путями. Практического смысла это не несет (примерно таким же образом можно решать уравнения первой/второй степени любого (не обязательно общего) вида).

Целью этой статьи является, в частности, подогрев интереса к анализу мат. функций и вообще к математике.

С вами был Петр, спасибо за внимание!

Решение квадратных уравнений по формуле: алгоритм решения

 

Квадратным уравнением называют уравнение вида a*x^2 +b*x+c=0, где a,b,c некоторые произвольные вещественные (действительные) числа, а x – переменная.2-4*a*c.

2. В зависимости от значения дискриминанта вычислить корни по формулам:

D<0, корней нет.

D=0, x=(-b/(2*a)

D>0, x=(-b+√D)/(2*a), x=(-b-√D)/(2*a)

Данный алгоритм универсален и подходит для решения любых квадратных уравнений. Полных и не полных, приведенных и неприведенных.

Нужна помощь в учебе?



Предыдущая тема: Решение квадратных уравнений выделением квадрата двучлена
Следующая тема:&nbsp&nbsp&nbspРешение задач с помощью квадратных уравнений: алгоритм и примеры

Решение квадратных уравнений

Решение квадратных уравнений

Квадратное уравнение — это уравнение, которое можно записать как

ось 2 + bx + c = 0

, когда a 0.

Существует три основных метода решения квадратных уравнений: факторинг, использование формулы квадратиков и завершение квадрата.

Факторинг

Чтобы решить квадратное уравнение на множители,

  1. Поместите все члены с одной стороны от знака равенства, оставив ноль с другой стороны.

  2. Коэффициент

    .

  3. Установите каждый коэффициент равным нулю.

  4. Решите каждое из этих уравнений.

  5. Проверьте, подставив свой ответ в исходное уравнение.

Пример 1

Решить x 2 — 6 x = 16.

Следуя инструкциям,

x 2 — 6 x = 16 становится x 2 — 6 x — 16 = 0

Коэффициент

.

( x -8) ( x + 2) = 0

Установка каждого коэффициента на ноль,

Затем проверить,

Оба значения, 8 и –2, являются решениями исходного уравнения.

Пример 2

Решить y 2 = — 6 y — 5.

Устанавливая все члены равными нулю,

y 2 + 6 y + 5 = 0

Коэффициент

.

( y + 5) ( y + 1) = 0

Установка каждого коэффициента на 0,

Для проверки, y 2 = –6 y — 5

Квадратичный с отсутствующим членом называется неполным квадратичным элементом (при условии, что член ax 2 не пропущен).

Пример 3

Решить x 2 — 16 = 0.

Коэффициент

.

Для проверки, x 2 — 16 = 0

Пример 4

Решить x 2 + 6 x = 0.

Коэффициент

.

Для проверки, x 2 + 6 x = 0

Пример 5

Решить 2 x 2 + 2 x — 1 = x 2 + 6 x — 5.

Во-первых, упростите, поместив все термины в одну сторону и комбинируя одинаковые термины.

Теперь фактор.

Для проверки, 2 x 2 + 2 x — 1 = x 2 + 6 x — 5

Квадратичная формула

Многие квадратные уравнения не могут быть решены факторизацией. Обычно это верно, когда корни или ответы не являются рациональными числами. Второй метод решения квадратных уравнений включает использование следующей формулы:

a, b, и c взяты из квадратного уравнения, записанного в его общем виде

ось 2 + bx + c = 0

, где a — это число перед x 2 , b — это число перед x , а c — это число без переменной рядом с ним (a .k.a., «постоянная»).

При использовании формулы корней квадратного уравнения вы должны знать о трех возможностях. Эти три возможности различаются частью формулы, называемой дискриминантом. Дискриминант — это значение под знаком корня, b 2 — 4 ac . Квадратное уравнение с действительными числами в качестве коэффициентов может иметь следующее:

  1. Два разных действительных корня, если дискриминант b 2 -4 ac является положительным числом.

  2. Один действительный корень, если дискриминант b 2 -4 ac равен 0.

  3. Нет действительного корня, если дискриминант b 2 -4 ac является отрицательным числом.

Пример 6

Решите относительно x : x 2 — 5 x = –6.

Устанавливаем все члены равными 0,

x 2 -5 x + 6 = 0

Затем замените 1 (который, как предполагается, стоит перед x 2 ), –5 и 6 вместо a , b и c, соответственно в формуле корней квадратного уравнения и упростите.

Поскольку дискриминант b 2 — 4 ac положительный, вы получаете два разных действительных корня.

Пример производит рациональные корни. В примере , квадратная формула используется для решения уравнения, корни которого нерациональны.

Пример 7

Решить относительно y : y 2 = –2y + 2.

Устанавливаем все члены равными 0,

y 2 + 2 y — 2 = 0

Затем замените 1, 2 и –2 на a , b и c, соответственно в формуле корней квадратного уравнения и упростите.

Обратите внимание, что два корня иррациональны.

Пример 8

Решить относительно x : x 2 + 2 x + 1 = 0.

Подставляя в формулу корней квадратного уравнения,

Поскольку дискриминант b 2 -4 ac равен 0, уравнение имеет один корень.

Квадратичная формула также может использоваться для решения квадратных уравнений, корни которых являются мнимыми числами, то есть они не имеют решения в действительной системе счисления.

Пример 9

Найдите x : x ( x + 2) + 2 = 0 или x 2 + 2 x + 2 = 0.

Подставляя в формулу корней квадратного уравнения,

Поскольку дискриминант b 2 — 4 ac отрицателен, это уравнение не имеет решения в действительной системе счисления.

Но если бы вы выразили решение с помощью мнимых чисел, решения были бы такими.

Завершение квадрата

Третий метод решения квадратных уравнений, который работает как с действительными, так и с мнимыми корнями, называется завершением квадрата.

  1. Запишите уравнение в виде ax 2 + bx = — c .

  2. Убедитесь, что a = 1 (если a 1, умножьте уравнение на, прежде чем продолжить).

  3. Используя значение b из этого нового уравнения, добавьте к обеим сторонам уравнения, чтобы получить полный квадрат в левой части уравнения.

  4. Найдите квадратный корень из обеих частей уравнения.

  5. Решите полученное уравнение.

Пример 10

Решите для x : x 2 — 6 x + 5 = 0.

Оформить в виде

Поскольку a = 1, прибавьте или 9 к обеим сторонам, чтобы завершить квадрат.

Извлеките квадратный корень из обеих частей.

x — 3 = ± 2

Решить.

Пример 11

Решить относительно y : y 2 + 2 y — 4 = 0.

Оформить в виде

Поскольку a = 1, прибавьте или 1 к обеим сторонам, чтобы завершить квадрат.

Извлеките квадратный корень из обеих частей.

Решить.

Пример 12

Решите относительно x : 2 x 2 + 3 x + 2 = 0.

Оформить в виде

Поскольку a ≠ 1, умножаем уравнение на.

Добавьте или с обеих сторон.

Извлеките квадратный корень из обеих частей.

В действительной системе счисления нет решения. Вам может быть интересно узнать, что завершение квадратного процесса для решения квадратных уравнений использовалось в уравнении ax 2 + bx + c = 0 для вывода формулы квадратичного уравнения.

Решите квадратное уравнение с помощью программы «Пошаговое решение математических задач»

Решение уравнений — центральная тема алгебры. Все приобретенные навыки в конечном итоге приводят к способности решать уравнения и упрощать решения. В предыдущих главах мы решали уравнения первой степени. Теперь у вас есть необходимые навыки для решения уравнений второй степени, которые известны как квадратных уравнений .

КВАДРАТИКА, РЕШЕННАЯ ФАКТОРИНГОМ

ЗАДАЧИ

По завершении этого раздела вы сможете:

  1. Определите квадратное уравнение.
  2. Приведите квадратное уравнение в стандартную форму.
  3. Решите квадратное уравнение, вычисляя множители.

Квадратное уравнение — это полиномиальное уравнение, которое содержит вторую, но не более высокую степень переменной.

Стандартная форма квадратного уравнения — ax 2 + bx + c = 0, когда a ≠ 0 и a, b и c — действительные числа.

Все квадратные уравнения могут быть представлены в стандартной форме, и любое уравнение, которое может быть преобразовано в стандартную форму, является квадратным уравнением.Другими словами, стандартная форма представляет все квадратные уравнения.

Решение уравнения иногда называют корнем уравнения.

Эта теорема доказана в большинстве учебных пособий по алгебре.

Важная теорема, которую невозможно доказать на уровне этого текста, гласит: «Каждое полиномиальное уравнение степени n имеет ровно n корней». Использование этого факта говорит нам, что квадратные уравнения всегда будут иметь два решения.Возможно, что два решения равны.

Квадратное уравнение будет иметь два решения, поскольку оно имеет степень два.

Самый простой метод решения квадратичных вычислений — это факторизация. Этот метод не всегда можно использовать, потому что не все многочлены факторизуемы, но он используется всякий раз, когда факторизация возможна.

Метод решения с помощью факторизации основан на простой теореме.

Если AB = 0, то либо A = 0, либо B = 0.

Другими словами, если произведение двух множителей равно нулю, то по крайней мере один из множителей равен нулю.

Мы не будем пытаться доказывать эту теорему, но внимательно отметим, что в ней говорится. Мы никогда не сможем перемножить два числа и получить ответ ноль, если хотя бы одно из чисел не равно нулю. Конечно, оба числа могут быть нулевыми, поскольку (0) (0) = 0.

Решение Шаг 1 Приведите уравнение в стандартную форму.

Мы должны вычесть 6 с обеих сторон.

Шаг 2 Полностью разложите на множители.

Вспомните, как разложить на множители трехчлены.

Шаг 3 Установите каждый коэффициент равным нулю и решите относительно x. Поскольку у нас есть (x — 6) (x + 1) = 0, мы знаем, что x — 6 = 0 или x + 1 = 0, и в этом случае x = 6 или x = — 1.

Здесь применяется приведенная выше теорема, согласно которой хотя бы один из факторов должен иметь нулевое значение.

Шаг 4 Проверьте решение в исходном уравнении. Если x = 6, то x 2 — 5x = 6 становится

Проверка ваших решений — верный способ узнать, правильно ли вы решили уравнение.

Следовательно, x = 6 является решением. Если x = — 1, то x 2 — 5x = 6 становится

Следовательно, — 1 — решение.

Решения могут быть обозначены либо записью x = 6 и x = — 1, либо записью множества и записью {6, — 1}, что мы читаем: «набор решений для x равен 6 и — 1.«В этом тексте мы будем использовать обозначение набора.

В этом примере 6 и -1 называются элементами набора.

Обратите внимание, что в этом примере уравнение уже имеет стандартную форму.

Опять же, проверка решений убедит вас, что вы не допустили ошибки при решении уравнения.
также называют корнями уравнения.

(x + 1) — наименьший общий знаменатель всех дробей в уравнении.
Помните, что каждый член уравнения нужно умножить на (x + 1).

Проверьте решения в исходном уравнении.

Проверьте исходное уравнение, чтобы убедиться, что знаменатель не равен нулю.

Обратите внимание, что здесь два решения равны. Это происходит только тогда, когда трехчлен является полным квадратом.

НЕПОЛНАЯ КВАДРАТИКА

ЗАДАЧИ

По завершении этого раздела вы сможете:

  1. Определите неполное квадратное уравнение.
  2. Решите неполное квадратное уравнение.

Если, когда уравнение помещено в стандартную форму ax 2 + bx + c = 0, либо b = 0, либо c = 0, уравнение представляет собой неполное квадратичное .

Пример 1

5x 2 — 10 = 0 является неполным квадратичным, так как средний член отсутствует и, следовательно, b = 0.

Когда вы сталкиваетесь с неполной квадратичной с c — 0 (отсутствует третий член), ее все же можно решить с помощью факторизации.

x — общий множитель. Произведение двух факторов равно нулю. Поэтому мы используем теорему из предыдущего раздела.
Проверьте эти решения.

Обратите внимание, что если член c отсутствует, вы всегда можете множить x из других членов. Это означает, что во всех таких уравнениях нуль будет одним из решений.
Неполная квадратичная система с отсутствующим членом b должна быть решена другим методом, поскольку факторизация будет возможна только в особых случаях.

Пример 3 Решите относительно x, если x 2 — 12 = 0.

Решение Поскольку x 2 — 12 не имеет общего множителя и не является разностью квадратов, его нельзя разложить на рациональные множители. Но из предыдущих наблюдений мы имеем следующую теорему.

Обратите внимание, что есть два значения, которые в квадрате будут равны A.

Используя эту теорему, мы имеем


Проверьте эти решения.

Добавьте 10 с каждой стороны. Проверьте эти решения.

Здесь 7x — общий множитель. Проверьте эти решения.

Обратите внимание, что в этом примере у нас есть квадрат числа, равного отрицательному числу. Это никогда не может быть правдой в действительной системе счисления, и поэтому у нас нет реального решения.

ЗАВЕРШЕНИЕ ПЛОЩАДИ

ЗАДАЧИ

По завершении этого раздела вы сможете:

  1. Определите трехчлен полного квадрата.
  2. Завершите третий член, чтобы получить трехчлен в виде полного квадрата.
  3. Решите квадратное уравнение, заполнив квадрат.

Из вашего опыта факторинга вы уже понимаете, что не все многочлены факторизуемы. Следовательно, нам нужен метод решения квадратичных вычислений, которые не подлежат факторизации. Необходимый метод называется «завершение квадрата».

Сначала давайте рассмотрим значение «трехчлена полного квадрата». Когда мы возводим двучлен в квадрат, мы получаем полный квадрат трехчлена.Общая форма: (a + b) 2 = a 2 + 2ab + b 2 .

Помните, возведение бинома в квадрат означает его умножение на себя.

Из общей формы и этих примеров мы можем сделать следующие наблюдения относительно трехчлена полного квадрата.

  1. Два из трех членов — полные квадраты. 4x 2 и 9 в первом примере, 25x 2 и 16 во втором примере, а также 2 и b 2 в общем виде.
    Другими словами, первый и третий члены представляют собой полные квадраты.
  2. Другой член — это два плюс или минус произведение квадратных корней из двух других членов.

Член -7 сразу говорит, что это не может быть трехчлен полного квадрата. Задача при заполнении квадрата состоит в том, чтобы найти число, которое заменит -7 таким образом, чтобы получился идеальный квадрат.

Рассмотрим эту задачу: заполните пробел так, чтобы «x 2 + 6x + _______» было трехчленом в виде полного квадрата.Из двух условий для трехчлена полного квадрата мы знаем, что пробел должен содержать полный квадрат и что 6x должно быть удвоенным произведением квадратного корня x 2 и числа в пробеле. Поскольку x уже присутствует в 6x и представляет собой квадратный корень из x 2 , то 6 должно быть в два раза больше квадратного корня из числа, которое мы помещаем в пробел. Другими словами, если мы сначала возьмем половину 6, а затем возведем в квадрат этот результат, мы получим необходимое число для бланка.

Следовательно, x 2 + 6x + 9 — это трехчлен полного квадрата.

Теперь давайте рассмотрим, как мы можем использовать завершение квадрата для решения квадратных уравнений.

Пример 5 Решите x 2 + 6x — 7 = 0, завершив квадрат.

Напомним, что вместо -7, +9 сделает выражение идеальным квадратом.

Решение Сначала мы замечаем, что член -7 необходимо заменить, если мы хотим получить трехчлен в виде полного квадрата, поэтому мы перепишем уравнение, оставив пустое поле для нужного числа.

Здесь будьте осторожны, чтобы не нарушить никаких правил алгебры. Например, обратите внимание, что вторая форма появилась в результате добавления +7 к обеим сторонам уравнения. Никогда не добавляйте что-либо к одной стороне, не добавляя то же самое к другой стороне.

Теперь мы находим половину 6 = 3 и 3 2 = 9, чтобы получить число для пробела. Опять же, если мы поместим 9 в пустое поле, мы также должны добавить 9 к правой стороне.

Помните, что если 9 добавляется к левой части уравнения, это также должно быть добавлено к правой части.

Теперь разложите на множители трехчлена полного квадрата, что дает

Теперь x 2 + 6x + 9 можно записать как (x + 3) 2 .

Таким образом, 1 и -7 являются решениями или корнями уравнения.

Пример 6 Решите 2x 2 + 12x — 4 = 0, заполнив квадрат.

Решение Эта проблема порождает еще одну трудность.Первый член, 2x 2 , не является полным квадратом.
Мы исправим это, разделив все члены уравнения на 2 и получим

Другими словами, получите коэффициент 1 для члена x 2 .

Теперь прибавим 2 к обеим сторонам, получив


Опять же, это более лаконично.

Пример 7 Решите 3x 2 + 7x — 9 = 0, заполнив квадрат.

Решение Шаг 1 Разделите все члены на 3.

Опять же, получите коэффициент 1 для x 2 , разделив на 3.

Шаг 2 Перепишите уравнение, оставив пробел для члена, необходимого для завершения квадрата.

Шаг 3 Найдите квадрат половины коэффициента при x и прибавьте к обеим сторонам.

Это выглядит сложно, но мы следуем тем же правилам, что и раньше.

Шаг 4 Разложите квадрат на множители.

Факторинг никогда не должен быть проблемой, поскольку мы знаем, что у нас есть полный квадратный трехчлен, что означает, что мы находим квадратные корни из первого и третьего членов и используем знак среднего члена.

Если у вас возникнут какие-либо затруднения, вам следует еще раз повторить арифметику при сложении чисел справа.
Теперь у нас

Шаг 5 Извлеките квадратный корень из каждой части уравнения.

Шаг 6 Решите относительно x (два значения).

не может быть упрощено. Мы могли бы также записать решение этой проблемы в более сжатой форме как

Выполните шаги, описанные в предыдущем вычислении, а затем обратите особое внимание на последнее значение. Каков вывод, когда квадрат количества равен отрицательному числу? «Нет реального решения».

Какое действительное число мы можем возвести в квадрат и получить -7?

Таким образом, чтобы решить квадратное уравнение, заполнив квадрат, следуйте этому пошаговому методу.

Шаг 1 Если коэффициент при x2 не равен 1, разделите все члены на этот коэффициент.
Шаг 2 Перепишите уравнение в виде x2 + bx + _______ = c + _______.
Шаг 3 Найдите квадрат половины коэффициента члена x и прибавьте эту величину к обеим сторонам уравнения.
Шаг 4 Разложите заполненный квадрат на множители и сложите числа в правой части уравнения.
Шаг 5 Найдите квадратный корень из каждой части уравнения.
Шаг 6 Решите относительно x и упростите.
Если шаг 5 невозможен, уравнение не имеет реального решения.

Эти шаги помогут решить уравнения в следующем упражнении.

КВАДРАТИЧЕСКАЯ ФОРМУЛА

ЗАДАЧИ

По завершении этого раздела вы сможете:

  1. Решите общее квадратное уравнение, заполнив квадрат.
  2. Решите любое квадратное уравнение, используя формулу корней квадратного уравнения.
  3. Решите квадратное уравнение, заполнив квадрат.

Стандартная форма квадратного уравнения — ax 2 + bx + c = 0. Это означает, что каждое квадратное уравнение может быть представлено в этой форме. В некотором смысле ax 2 + bx + c = 0 представляет все квадратичные системы. Если вы сможете решить это уравнение, у вас будет решение всех квадратных уравнений.

Решим общее квадратное уравнение методом завершения квадрата.

Это необходимо для получения члена x 2 с коэффициентом 1.
Это мы делали в предыдущем разделе много раз.

Надо прибавить с каждой стороны.

Эта форма называется квадратной формулой и представляет собой решение всех квадратных уравнений.

Запомните это выражение.

Чтобы использовать формулу квадратного уравнения, вы должны указать a, b и c. Для этого данное уравнение всегда необходимо оформлять в стандартном виде.

Осторожно подставьте значения a, b и c в формулу.

Не каждое квадратное уравнение имеет реальное решение.

Это уравнение уже имеет стандартную форму.

Реального решения нет, так как -47 не имеет действительного квадратного корня.

Опять же, это уравнение в стандартной форме.

Теперь это решение следует упростить.

ПРОБЛЕМЫ СО СЛОВОМ

ЗАДАЧИ

По завершении этого раздела вы сможете:

  1. Определите текстовые задачи, для решения которых требуется квадратное уравнение.
  2. Решать текстовые задачи, связанные с квадратными уравнениями.

Некоторые типы словесных задач могут быть решены с помощью квадратных уравнений. Процесс обрисовки и постановки проблемы такой же, как описано в главе 5, но с проблемами, решаемыми квадратичными методами, вы должны быть очень осторожны, проверяя решения в самой проблеме. Физические ограничения внутри проблемы могут устранить одно или оба решения.

Пример 1 Если длина прямоугольника на 1 единицу больше, чем в два раза больше ширины, а его площадь составляет 55 квадратных единиц, найдите длину и ширину.

Решение Формула площади прямоугольника: Площадь = Длина X Ширина. Пусть x = ширина, 2x + 1 = длина.

Если x представляет ширину, то 2x представляет удвоенную ширину, а 2x + 1 представляет единицу более чем удвоенную ширину.

Приведите квадратное уравнение в стандартную форму.
Эта квадратичная величина может быть решена путем факторизации.

На этом этапе вы можете видеть, что решение x = -11/2 недействительно, поскольку x представляет собой измерение ширины, а отрицательные числа не используются для таких измерений.Следовательно, решение

ширина = x = 5, длина = 2x + 1 = 11.

Измерение не может быть отрицательным значением.

Значение x равно.
Помните, что ЖК-дисплей означает наименьший общий знаменатель.
Каждый член нужно умножить в 10 раз.
Опять же, эту квадратичную величину можно разложить на множители.

Оба решения проверяют. Следовательно, набор решений есть.

Есть два решения этой проблемы.

Пример 3 Если определенное целое число вычитается из его квадрата, умноженного на 6, получается 15. Найдите целое число.

Решение Пусть x = целое число. Тогда

Поскольку ни одно из решений не является целым числом, проблема не имеет решения.

У вас может возникнуть соблазн указать эти значения в качестве решения, если вы не обратили пристальное внимание на тот факт, что проблема запрашивала целое число.

Пример 4 Управляющий фермой имеет под рукой 200 метров забора и хочет оградить прямоугольное поле так, чтобы его площадь составляла 2400 квадратных метров.Какими должны быть размеры поля?

Решение Здесь задействованы две формулы. P = 2l + 2w для периметра и A = lw для площади.
Сначала используя P = 2l + 2w, получаем

Теперь мы можем использовать формулу A = lw и подставить (100 — l) вместо w, получив

Поле должно быть шириной 40 метров и длиной 60 метров.

Мы могли бы точно так же решить для l, получив l = 100 — w. Тогда

Обратите внимание, что в этой задаче мы фактически используем систему уравнений

P = 2 l + 2 w
A = l w.

В общем случае система уравнений, в которой участвует квадратичная функция, будет решаться методом подстановки. (См. Главу 6.)

РЕЗЮМЕ

Ключевые слова

  • Квадратное уравнение — это полиномиальное уравнение от одной неизвестной, которое содержит вторую степень, но не более высокую степень переменной.
  • Стандартная форма квадратного уравнения : ax 2 + bx + c = 0, когда a 0.
  • Неполное квадратное уравнение имеет вид ax 2 + bx + c = 0, и либо b = 0, либо c = 0.
  • Квадратичная формула равна

Процедуры

  • Самый прямой и, как правило, самый простой метод поиска решений квадратного уравнения — это факторизация. Этот метод основан на теореме: если AB = 0, то A = 0 или B = 0. Чтобы использовать эту теорему, мы приводим уравнение в стандартную форму, коэффициент и устанавливаем каждый коэффициент равным нулю.
  • Чтобы решить квадратное уравнение, заполнив квадрат, выполните следующие действия:
    Шаг 1 Если коэффициент при x 2 не равен 1, разделите все члены на этот коэффициент.
    Шаг 2 Перепишите уравнение в виде x 2 + bx + _____ = c + _____
    Шаг 3 Найдите квадрат половины коэффициента члена x и прибавьте эту величину к обеим частям. уравнения.
    Шаг 4 Разложите заполненный квадрат на множители и сложите числа в правой части уравнения.
    Шаг 5 Найдите квадратный корень из каждой части уравнения.
    Шаг 6 Решите относительно x и упростите.
  • Метод завершения квадрата используется для вывода формулы корней квадратного уравнения.
  • Чтобы использовать квадратную формулу, напишите уравнение в стандартной форме, укажите a, b и c и подставьте эти значения в формулу. Все решения следует упростить.

Обзор различных методов решения квадратного уравнения — Концепция

Решение квадратных уравнений может быть трудным, но, к счастью, есть несколько различных методов, которые мы можем использовать в зависимости от того, какой тип квадратичного уравнения мы пытаемся решить.Четыре метода решения квадратного уравнения — факторизация с использованием квадратных корней, завершение квадрата и квадратной формулы.

Итак, сейчас я хочу поговорить об обзоре всех различных способов решения квадратного уравнения. Под этим я подразумеваю что-нибудь в форме: ax² плюс bx плюс c. Итак, у нас есть четыре различных способа, которые нам удобнее.У нас есть факторизация, свойство извлечения квадратного корня, завершение квадрата и квадратная формула. Мы можем использовать эти методы в разное время, и я просто хочу поговорить о том, когда мы можем их использовать, почему они хороши и почему плохие. Так что я просто спущусь вниз по ряду и расскажу о каждом из них. «Чек» означает «за», а «минус» — «против». Факторинг обычно является самым быстрым и простым способом решения чего-либо, когда это возможно. Часто мы имеем дело с квадратичным коэффициентом, который нельзя факторизовать, поэтому факторинг нам не поможет.Так что это быстро и просто, когда его можно использовать, но также не всегда можно использовать. Так быстро и просто, но не всегда применимо.
Следующее, о чем мы поговорим, — это свойство квадратного корня. Это когда у нас есть что-то квадратное. Итак, профи: это здорово, когда вы решаете что-то квадратное. Единственная проблема в том, что мы не всегда имеем дело с ситуацией. Каждый раз, когда у вас есть X-термин или что-то в этом роде, мы не сможем его использовать. Так что это не всегда квадратный термин.Когда это применимо, это здорово, но не всегда. На самом деле это не так часто бывает.
Завершение пл. Самое замечательное в завершении квадрата — это то, что мы всегда можем это сделать. Никогда не будет времени, когда вы не сможете завершить квадрат. Но недостаток в том, что это может стать некрасивым. Если вы имеете дело с коэффициентом или нечетным средним членом или чем-то в этом роде, вы собираетесь ввести дроби. Это не всегда лучшая ситуация.
И, наконец, формула корней квадратного уравнения.Опять же, это здорово, потому что им всегда можно воспользоваться. И минусы, это зависит от человека. Если вы используете квадратные корни, что не всегда нравится некоторым людям, вам всегда нужно использовать квадратные корни. Обычно это не так просто, как некоторые из этих других методов, я бы сказал, что завершение квадрата немного проще, но это то, что вы должны запомнить. Поэтому вам нужно запомнить формулу, и она может стать некрасивой.
Итак, это четыре разных способа, плюсы и минусы, а также некоторые вещи, о которых следует подумать при решении проблемы.2} + bx + c = 0, потому что трехчлен в левой части нелегко вынести за скобки. Это не означает, что квадратное уравнение не имеет решения. На этом этапе нам нужно обратиться к прямому подходу квадратной формулы, чтобы найти решения квадратного уравнения или, проще говоря, определить значения x, которые могут удовлетворять уравнению.

Чтобы использовать квадратную формулу, квадратное уравнение, которое мы решаем, необходимо преобразовать в «стандартную форму», в противном случае все последующие шаги не будут работать. 2} + bx + c = 0.

При необходимости снизьте скорость. Будьте осторожны с каждым шагом, упрощая выражения. Здесь обычно случаются типичные ошибки, потому что учащиеся склонны «расслабляться», что приводит к ошибкам, которые можно было предотвратить, например, при сложении, вычитании, умножении и / или делении действительных чисел.


Примеры решения квадратных уравнений по квадратичной формуле

Пример 1 : Решите квадратное уравнение ниже, используя квадратную формулу.

При осмотре очевидно, что квадратное уравнение имеет стандартную форму, поскольку правая часть равна нулю, а остальные члены остаются в левой части. Другими словами, у нас есть что-то вроде этого

Это здорово! Нам нужно просто определить значения a, b и c, а затем подставить их в формулу корней квадратного уравнения.

Вот и все! Возьмите за привычку всегда проверять решенные значения x обратно в исходное уравнение.


Пример 2 : Решите квадратное уравнение ниже, используя квадратную формулу.

Это квадратное уравнение абсолютно не в той форме, в которой мы хотим, потому что правая часть НЕ ноль. Мне нужно удалить это 7 с правой стороны, вычтя обе части на 7. Это решит нашу проблему. После этого решите относительно x как обычно.

Окончательные ответы: {x_1} = 1 и {x_2} = — {2 \ over 3}.


Пример 3 : Решите квадратное уравнение ниже, используя квадратную формулу.

Это квадратное уравнение выглядит как «беспорядок».У меня есть переменные x и константы по обе стороны уравнения. Если мы сталкиваемся с чем-то подобным, всегда придерживайтесь того, что мы знаем. Да, все дело в стандартной форме. Мы должны заставить правую часть равняться нулю. Мы можем сделать это за два шага.

Сначала я вычту обе части на 5x, а затем прибавлю 8.

Нам нужны:

a = — 1, b = — \, 8 и c = 2


Пример 4 : Решите квадратное уравнение ниже, используя квадратную формулу.

Что ж, если вы думаете, что Пример 3 — это «беспорядок», тогда он должен быть еще более «беспорядочным». Однако вскоре вы поймете, что они действительно очень похожи.

Сначала нам нужно выполнить некоторую очистку, преобразовав это квадратное уравнение в стандартную форму. Звучит знакомо? Поверьте, эта проблема не так плоха, как кажется, если мы знаем, что делать.

Напоминаю, что нам нужно что-то вроде этого

Следовательно, мы должны сделать все возможное, чтобы правая часть уравнения стала равной нулю.2} термин справа.

  • Удалите член x с правой стороны.
  • Удалите константу с правой стороны.

После получения правильной стандартной формы на предыдущем шаге теперь пора подставить значения a, b и c в формулу корней квадратного уравнения, чтобы найти x.

  • Из преобразованной стандартной формы извлеките требуемые значения.

a = 1, b = — \, 4 и c = — \, 14

  • Затем вычислите эти значения в формуле корней квадратного уравнения.

Практика с рабочими листами


Возможно, вас заинтересует:

Решение квадратных уравнений методом квадратного корня
Решение квадратных уравнений методом факторинга
Решение квадратных уравнений путем заполнения квадрата

Решите квадратные уравнения по квадратичной формуле — элементарная алгебра

Цели обучения

К концу этого раздела вы сможете:

  • Решите квадратные уравнения, используя формулу корней квадратного уравнения
  • Используйте дискриминант, чтобы предсказать количество решений квадратного уравнения
  • Определите наиболее подходящий метод решения квадратного уравнения

Прежде чем начать, пройдите тест на готовность.

  1. Упростить:.
    Если вы пропустили эту проблему, просмотрите (рисунок).
  2. Упростить:.
    Если вы пропустили эту проблему, просмотрите (рисунок).
  3. Упростить:.
    Если вы пропустили эту проблему, просмотрите (рисунок).

Когда мы решали квадратные уравнения в последнем разделе, завершая квадрат, мы каждый раз предпринимали одни и те же шаги. К концу набора упражнений вы, возможно, задавались вопросом: «А нет ли более простого способа сделать это?» Ответ — «да». В этом разделе мы выведем и воспользуемся формулой, чтобы найти решение проблемы. квадратное уровненеие.

Мы уже видели, как решить формулу для конкретной переменной «в целом», чтобы мы проделали алгебраические шаги только один раз, а затем использовали новую формулу, чтобы найти значение конкретной переменной. Теперь мы рассмотрим этапы завершения квадрата в целом, чтобы решить квадратное уравнение для x . Возможно, будет полезно взглянуть на один из примеров в конце последнего раздела, где мы решали уравнение формы, когда вы читаете алгебраические шаги ниже, поэтому вы видите их как с числами, так и со словом «в целом».’

Последнее уравнение — квадратичная формула.

Квадратичная формула

Решения квадратного уравнения вида даются формулой:

Чтобы использовать квадратичную формулу, мы подставляем значения в выражение в правой части формулы. Затем мы делаем все математические вычисления, чтобы упростить выражение. Результат дает решение (я) квадратного уравнения.

Как решить квадратное уравнение с помощью квадратной формулы

Решите, используя дискриминант.

Решите, используя дискриминант.

Решите, используя дискриминант.

Если вы произносите формулу во время написания каждой задачи, вы быстро запомните ее. И помните, квадратная формула — это уравнение. Обязательно начинайте с «».

Решите, используя дискриминант.

Решите, используя дискриминант.

Решите, используя дискриминант.

Когда мы решали квадратные уравнения с помощью свойства квадратного корня, мы иногда получали ответы с радикалами. То же самое может случиться и при использовании квадратичной формулы. Если в качестве решения мы получаем радикал, окончательный ответ должен иметь радикал в его упрощенной форме.

Решите, используя дискриминант.

Решение

Мы можем использовать квадратичную формулу, чтобы найти переменную в квадратном уравнении, независимо от того, называется ли оно « x ».

Решите, используя дискриминант.

Решите, используя дискриминант.

Решите, используя дискриминант.

Решите, используя дискриминант.

Решите, используя дискриминант.

Мы не можем извлечь квадратный корень из отрицательного числа. Итак, когда мы подставляем, и в квадратную формулу, если величина внутри радикала отрицательна, квадратное уравнение не имеет реального решения.Мы увидим это в следующем примере.

Решите, используя дискриминант.

Решите, используя дискриминант.

Решите, используя дискриминант.

Все квадратные уравнения, которые мы решили до сих пор в этом разделе, были записаны в стандартной форме,. Иногда нам нужно сделать некоторую алгебру, чтобы привести уравнение в стандартную форму, прежде чем мы сможем использовать квадратичную формулу.

Решите, используя дискриминант.

Решите, используя дискриминант.

Решите, используя дискриминант.

Когда мы решали линейные уравнения, если в уравнении было слишком много дробей, мы «очищали дроби», умножая обе части уравнения на ЖК-дисплей. Это дало нам возможность решить эквивалентное уравнение — без дробей. Мы можем использовать ту же стратегию с квадратными уравнениями.

Решите, используя дискриминант.

Решите, используя дискриминант.

Решите, используя дискриминант.

Подумайте об уравнении. Мы знаем из принципа нулевого произведения, что это уравнение имеет только одно решение:.

В следующем примере мы увидим, как использование квадратичной формулы для решения уравнения с полным квадратом также дает только одно решение.

Решите, используя дискриминант.

Решение

Вы узнали, что это идеальный квадрат?

Решите, используя дискриминант.

Решите, используя дискриминант.

Использование дискриминанта для предсказания числа решений квадратного уравнения

Когда мы решали квадратные уравнения в предыдущих примерах, иногда мы получали два решения, иногда одно решение, иногда нет реальных решений. Есть ли способ предсказать количество решений квадратного уравнения, не решая его на самом деле?

Да, количество внутри корня квадратной формулы позволяет нам легко определить количество решений.Эта величина называется дискриминантом.

Дискриминант

В квадратичной формуле величина называется дискриминантом.

Давайте посмотрим на дискриминант уравнений на (Рисунок), (Рисунок) и (Рисунок), а также на количество решений этих квадратных уравнений.

Когда дискриминант положительный квадратное уравнение имеет два решения .

Когда дискриминант ноль квадратное уравнение имеет одно решение .

Когда дискриминант отрицательный , квадратное уравнение не имеет реальных решений .

Определите количество решений каждого квадратного уравнения:

ⓐⓑⓒⓓ

ⓐ нет реальных решений ⓑ 2 ⓒ 1 ⓓ нет реальных решений

Определите количество решений каждого квадратного уравнения:

ⓐⓑⓒⓓ

ⓐ 2 ⓑ нет реальных решений ⓒ 1 ⓓ 2

Определите наиболее подходящий метод для решения квадратного уравнения

Мы использовали четыре метода для решения квадратных уравнений:

  • Факторинг
  • Свойство квадратного корня
  • Завершение площади
  • Квадратичная формула

Вы можете решить любое квадратное уравнение, используя квадратную формулу, но это не всегда самый простой метод.

Определите наиболее подходящий метод решения квадратного уравнения.

  1. Попробуйте сначала Факторинг . Если квадратичные множители легко, этот метод очень быстрый.
  2. Далее попробуйте свойство квадратного корня . Если уравнение соответствует форме или, его можно легко решить с помощью свойства квадратного корня.
  3. Используйте квадратную формулу . Любое квадратное уравнение можно решить с помощью квадратной формулы.

А как насчет метода завершения квадрата? Большинство людей считают этот метод громоздким и предпочитают не использовать его.Нам нужно было включить его в эту главу, потому что мы завершили квадрат в целом, чтобы получить квадратную формулу. Вы также будете использовать процесс завершения квадрата в других областях алгебры.

Определите наиболее подходящий метод для решения каждого квадратного уравнения:

ⓐⓑⓒ

Решение

Так как уравнение находится в, наиболее подходящим методом является использование свойства квадратного корня.

Мы понимаем, что левая часть уравнения представляет собой трехчлен полного квадрата, поэтому факторинг будет наиболее подходящим методом.

Приведите уравнение в стандартную форму.

В то время как наша первая мысль может заключаться в том, чтобы попробовать факторинг, размышления обо всех возможностях проб и ошибок приводят нас к выбору квадратичной формулы как наиболее подходящего метода

Определите наиболее подходящий метод для решения каждого квадратного уравнения:

ⓐⓑⓒ

коэффициент ⓑ Свойство квадратного корня ⓒ Квадратичная формула

Определите наиболее подходящий метод для решения каждого квадратного уравнения:

ⓐⓑⓒ

ⓐ Квадратичная формула ⓑ факторинг ⓒ Свойство квадратного корня

Практика ведет к совершенству

Решите квадратные уравнения с помощью квадратичной формулы

В следующих упражнениях решите, используя квадратичную формулу.

Использование дискриминанта для прогнозирования числа решений квадратного уравнения

В следующих упражнениях определите количество решений каждого квадратного уравнения.

ⓐ нет реальных решений ⓑ 1
ⓒ 2 ⓓ нет реальных решений

ⓐ 1 ⓑ нет реальных решений
ⓒ 1 ⓓ 2

Определите наиболее подходящий метод для решения квадратного уравнения

В следующих упражнениях определите наиболее подходящий метод (разложение на множители, квадратный корень или квадратная формула) для решения каждого квадратного уравнения. Не решайте.

коэффициент ⓑ квадратный корень
ⓒ Квадратичная формула

коэффициент ⓑ квадратный корень
коэффициент

Повседневная математика

Ракета запускается прямо с корабля в море.Решите уравнение для количества секунд, в течение которых ракета будет находиться на высоте 640 футов.

Архитектор проектирует холл гостиницы. Она хочет иметь треугольное окно, выходящее в атриум, с шириной окна на 6 футов больше высоты. Из-за ограничений по энергопотреблению площадь окна должна составлять 140 квадратных футов. Решите уравнение для высоты окна.

Письменные упражнения

Решите уравнение
ⓐ, заполнив квадрат
ⓑ с помощью квадратичной формулы
ⓒ Какой метод вы предпочитаете? Почему?

ⓐⓑ
ⓒ ответы будут отличаться

Решите уравнение
ⓐ, заполнив квадрат
ⓑ с помощью квадратичной формулы
ⓒ Какой метод вы предпочитаете? Почему?

Самопроверка

ⓐ После выполнения упражнений используйте этот контрольный список, чтобы оценить свое мастерство в достижении целей этого раздела.

ⓑ Что этот контрольный список говорит вам о вашем мастерстве в этом разделе? Какие шаги вы предпримете для улучшения?

Глоссарий

дискриминант
В квадратичной формуле величина называется дискриминантом.

Калькулятор квадратичных формул

Калькулятор ниже решает квадратное уравнение

ax 2 + bx + c = 0

.

В алгебре квадратное уравнение — это любое полиномиальное уравнение второй степени следующего вида:

топор 2 + bx + c = 0

, где x — неизвестное значение, a называется квадратичным коэффициентом, b линейным коэффициентом и c константой.Цифры a , b и c являются коэффициентами уравнения, и они представляют известные числа. Например, a не может быть 0, иначе уравнение будет линейным, а не квадратичным. Квадратное уравнение можно решить несколькими способами, включая: факторинг, использование формулы квадратичного уравнения, завершение квадрата или построение графика. Здесь будет обсуждаться только использование квадратной формулы, а также основы завершения квадрата (поскольку вывод формулы включает завершение квадрата).Ниже представлена ​​квадратичная формула, а также ее вывод.

Вывод квадратичной формулы

С этого момента можно завершить квадрат, используя соотношение:

x 2 + bx + c = (x — h) 2 + k

Продолжение деривации с использованием этого отношения:

Напомним, что ± существует как функция вычисления квадратного корня, что дает решения квадратного уравнения как с положительными, так и с отрицательными корнями.Значения x , найденные с помощью квадратной формулы, являются корнями квадратного уравнения, которые представляют значения x , где любая парабола пересекает ось x. Кроме того, квадратная формула также обеспечивает ось симметрии параболы. Это демонстрирует приведенный ниже график. Обратите внимание, что квадратная формула на самом деле имеет множество реальных приложений, таких как вычисление площадей, траекторий снарядов и скорости, среди прочего.

квадратных уравнений | Решенные задачи и практические вопросы

В этой статье мы рассмотрим квадратные уравнения — определения, форматы, решенные задачи и примеры вопросов для практики.

Квадратное уравнение — это многочлен, наибольшая степень которого равна квадрату переменной (x 2 , y 2 и т. Д.)

Определения

Моном — это алгебраическое выражение, содержащее только один член.

Пример: x 3 , 2x, y 2 , 3xyz и т. Д.

Многочлен — это алгебраическое выражение, содержащее более одного члена.

В качестве альтернативы можно указать —

Многочлен формируется путем сложения / вычитания нескольких одночленов.

Пример: x 3 + 2y 2 + 6x + 10, 3x 2 + 2x-1, 7y-2 и т. Д.

Многочлен, содержащий два члена, называется биномиальным выражением .

Многочлен, содержащий три члена, называется выражением трехчлена .

Стандартное квадратное уравнение выглядит так:

топор 2 + bx + c = 0

Где a, b, c — числа и a≥1.

a, b называются коэффициентами x, , 2, и x соответственно, а c называется константой.

Ниже приведены примеры некоторых квадратных уравнений:

1) x 2 + 5x + 6 = 0, где a = 1, b = 5 и c = 6.

2) x 2 + 2x-3 = 0, где a = 1, b = 2 и c = -3

3) 3x 2 + 2x = 1

→ 3x 2 + 2x-1 = 0, где a = 3, b = 2 и c = -1

4) 9x 2 = 4

→ 9x 2 -4 = 0, где a = 9, b = 0 и c = -4

Для каждого квадратного уравнения может быть одно или несколько решений.Они называются корнями квадратного уравнения.

Для квадратного уравнения ax 2 + bx + c = 0,

сумма его корней = –b / a и произведение его корней = c / a.

Квадратное уравнение может быть выражено как произведение двух биномов.

Например, рассмотрим следующее уравнение

x 2 — (a + b) x + ab = 0

x 2 -ax-bx + ab = 0

х (х-а) -b (х-а) = 0

(х-а) (х-б) = 0

x-a = 0 или x-b = 0
x = a или x = b

Здесь a и b называются корнями данного квадратного уравнения.

Теперь давайте вычислим корни уравнения x 2 + 5x + 6 = 0.

Мы должны взять два числа, сложить которые мы получим 5 и умножить получим 6. Это 2 и 3.

Выразим средний член как сложение 2х и 3х.

→ х 2 + 2х + 3х + 6 = 0

→ х (х + 2) +3 (х + 2) = 0

→ (х + 2) (х + 3) = 0

→ x + 2 = 0 или x + 3 = 0

→ x = -2 или x = -3

Этот метод называется факторингом .

Ранее мы видели, что сумма корней равна –b / a, а произведение корней равно c / a. Давайте проверим это.

Сумма корней уравнения x 2 + 5x + 6 = 0 равна -5, а произведение корней равно 6.

Корни этого уравнения -2 и -3 при сложении дают -5, а при умножении дают 6.

Решенные примеры квадратных уравнений

Решим еще несколько примеров этим методом.

Задача 1: Решить для x: x 2 -3x-10 = 0

Решение :

Выразим -3x как сумму -5x и + 2x.

→ х 2 -5x + 2x-10 = 0

→ х (х-5) +2 (х-5) = 0

→ (х-5) (х + 2) = 0

→ x-5 = 0 или x + 2 = 0

→ x = 5 или x = -2

Задача 2: Решить относительно x: x 2 -18x + 45 = 0

Решение :

Числа, которые в сумме дают -18 и дают +45 при умножении: -15 и -3.

Переписывая уравнение,

→ х 2 -15x-3x + 45 = 0

→ х (х-15) -3 (х-15) = 0

→ (х-15) (х-3) = 0

→ x-15 = 0 или x-3 = 0

→ x = 15 или x = 3

До сих пор коэффициент x 2 был равен 1.Давайте посмотрим, как решить уравнения, в которых коэффициент при x 2 больше 1.

Задача 3: Решить относительно x: 3x 2 + 2x = 1

Решение :

Переписывая наше уравнение, получаем 3x 2 + 2x-1 = 0

Здесь коэффициент при x 2 равен 3. В этих случаях мы умножаем константу c на коэффициент x 2 . Следовательно, произведение выбранных нами чисел должно быть равно -3 (-1 * 3).

Выражение 2x как суммы + 3x и –x

→ 3x 2 + 3x-x-1 = 0

→ 3х (х + 1) -1 (х + 1) = 0

→ (3x-1) (x + 1) = 0

→ 3x-1 = 0 или x + 1 = 0

→ x = 1/3 или x = -1

Задача 4: Решите для x: 11x 2 + 18x + 7 = 0

Решение :

В этом случае сумма выбранных чисел должна быть равна 18, а произведение чисел должно быть равно 11 * 7 = 77.

Это можно сделать, представив 18x как сумму 11x и 7x.

→ 11x 2 + 11x + 7x + 7 = 0

→ 11x (x + 1) +7 (x + 1) = 0

→ (х + 1) (11x + 7) = 0

→ x + 1 = 0 или 11x + 7 = 0

→ x = -1 или x = -7/11.

Факторинг — простой способ найти корни. Но этот метод применим только к уравнениям, которые можно разложить на множители.

Например, рассмотрим уравнение x 2 + 2x-6 = 0.

Если мы возьмем +3 и -2, их умножение даст -6, но их сложение не даст +2. Следовательно, это квадратное уравнение нельзя разложить на множители.

Для этого вида уравнений мы применяем формулу корней квадратного уравнения, чтобы найти корни.

Квадратичная формула для нахождения корней,

x = [-b ± √ (b 2 -4ac)] / 2a

Теперь давайте найдем корни приведенного выше уравнения.

x 2 + 2x-6 = 0

Здесь a = 1, b = 2 и c = -6.

Подставляя эти значения в формулу,

x = [-2 ± √ (4 — (4 * 1 * -6))] / 2 * 1

→ x = [-2 ± √ (4 + 24)] / 2

→ x = [-2 ± √28] / 2

Когда мы получаем неполный квадрат в квадратном корне, мы обычно пытаемся выразить его как произведение двух чисел, одно из которых является точным квадратом. Это сделано для упрощения. Здесь 28 может быть выражено как произведение 4 и 7.

→ x = [-2 ± √ (4 * 7)] / 2

→ x = [-2 ± 2√7] / 2

→ x = 2 [-1 ± √7] / 2

→ х = -1 ± √7

Следовательно, √7-1 и -√7-1 являются корнями этого уравнения.

Рассмотрим другой пример.

Решить относительно x: x 2 = 24 — 10x

Решение :

Переписывая уравнение в стандартную квадратичную форму,

x 2 + 10x-24 = 0

Какие два числа при сложении дают +10, а при умножении — -24? 12 и -2.

Значит, это можно решить методом факторинга. Но давайте решим его новым методом, применив формулу корней квадратного уравнения.

Здесь a = 1, b = 10 и c = -24.

x = [-10 ± √ (100 — 4 * 1 * -24)] / 2 * 1

x = [-10 ± √ (100 — (- 96))] / 2

x = [-10 ± √196] / 2

x = [-10 ± 14] / 2

x = 2 или x = -12 — корни.

Дискриминант

Для уравнения ax 2 + bx + c = 0, b 2 -4ac называется дискриминантом и помогает в определении природы корней квадратного уравнения.

Если b 2 -4ac> 0, корни действительны и различны.

Если b 2 -4ac = 0, корни действительны и равны.

Если b 2 -4ac <0, корни не являются действительными (они комплексные).

Рассмотрим следующий пример:

Задача: Найдите характер корней уравнения x 2 + x + 12 = 0.

Решение :

b 2 -4ac = -47 для этого уравнения. Итак, у него сложные корни. Давайте проверим это.

→ [-1 ± √ (1-48)] / 2 (1)

→ [-1 ± √-47] / 2

√-47 обычно записывается как i √47, указывая на то, что это мнимое число.

Значит проверено.

Викторина по квадратным уравнениям: решите следующие

Проблема 1

Решить относительно x: x 2 -15x + 56 = 0

A. x = 14 или x = 4
B. x = 8 или x = 7
C. x = 28 или x = 2
D. Все вышеперечисленное

Ответ 1

Б.

Пояснение

Только 8 и 7 удовлетворяют условиям сложения 15 и получения произведения 56.

Задача 2

Найти x, если 2x 2 + 7x + 4 = 0

A. -7 ± √17 / 4
B. -7 ± √7 / 4
C. [-7 ± √17] / 4
D. [-7 ± √17] / 2

Ответ 2

C.

Пояснение :

Применяя формулу корней квадратного уравнения и подставляя a = 2, b = 7 и c = 4, мы получаем ответ как C.

Задача 3

При каком значении k уравнение x 2 -12x + k = 0 имеет действительные и равные корни?
А.6
Б. 35
С. 12
Д. 36

Ответ 3

Д.

Пояснение

b 2 -4ac = 0, чтобы уравнение имело действительные и равные корни.
144-4k = 0 → k = 36


Пройдите этот доступный онлайн-курс по квадратным уравнениям. Из решения, построения графиков и записи квадратного уравнения вы узнаете все шаг за шагом.